Эта статья является первой частью конспекта книги «Наша математическая вселенная. В поисках фундаментальной природы реальности» (автор Макс Тегмарк). Материал статьи посвящен многомировой интерпретации квантовой механики.

Является ли квантовая механика внутренне противоречивой? Действительно ли волновая функция коллапсирует? Если да, то когда? А если нет, то почему мы не видим вещи в двух местах сразу? Откуда появляются случайности и вероятности в квантовой механике?

В 1957 году принстонский аспирант Хью Эверетт предложил поистине радикальный ответ, подразумевающий существование параллельных вселенных. Однако эту идею в основном игнорировали. В чем же идея Эверетта? Это на удивление простое утверждение: Волновая функция не коллапсирует. Никогда. Иными словами, волновая функция, которая полностью описывает нашу Вселенную, всегда изменяется детерминистически, всегда подчиняется уравнению Шредингера, независимо от того, выполняются наблюдения или нет.

Представление о том, что в некоторые волшебные моменты реальность испытывает своего рода метафизическое расщепление на две ветви, которые в дальнейшем никогда не взаимодействуют, – это ошибочное изложение диссертации Эверетта. Оно, кроме того, противоречит его постулату о том, что волновая функция никогда не коллапсирует, поскольку дальнейшее развитие может, в принципе, заставить эти ветви интерферировать друг с другом. Согласно Эверетту, была, есть и всегда будет лишь одна волновая функция, и только расчеты декогеренции, а не постулаты, могут показать, когда с хорошим приближением можно рассматривать две ветви как не взаимодействующие.

Эверетт не выдвигал предположений ни о каких безумных вещах, но все они возникали в качестве следствий из его предположений. В конце концов, Нильс Бор и его коллеги изобрели коллапс волновой функции, чтобы объяснить, почему в экспериментах мы видим тот или иной исход. Но Эверетт понял: даже если у экспериментов нет определенных исходов, они могут выглядеть так, будто такие исходы у них есть.

Рис. 1 – Мысленный эксперимент с «квантовыми картами».
Рис. 1 – Мысленный эксперимент с «квантовыми картами».

В мысленном эксперименте «квантовые карты» (рис. 1) вы берете карту с идеально ровным нижним краем, устанавливаете ее на столе вертикально и ставите на то, что она, падая, ляжет лицом вверх.

На практике эта неустойчивая карта, конечно, вскоре упадет из-за едва заметного движения воздуха, так что лучше взять нормальную карту с толстым нижним краем и использовать квантовое устройство вроде шредингеровского триггера с радиоактивным атомом, чтобы подтолкнуть ее в ту или иную сторону.

Согласно уравнению Шредингера, она упадет в течение нескольких секунд, даже если вы приложите все силы, чтобы идеально ее сбалансировать, поскольку принцип неопределенности Гейзенберга утверждает, что она не может находиться лишь в одном положении (строго вертикальном) без всякого движения. Но если исходное состояние было симметричным относительно направления вправо-влево, то таким должно быть и окончательное состояние. Это подразумевает, что карта упадет в обе стороны сразу, в суперпозиции.

Когда вы открываете глаза и смотрите на карту, вы совершаете наблюдение. Согласно копенгагенской интерпретации, волновая функция коллапсирует, и вы видите карту лежащей либо лицом вверх, либо лицом вниз с вероятностью 50 % для каждого исхода. А что говорит Эверетт? Для него в наблюдении нет ничего магического: это просто физический процесс, связанный с передачей информации – в данном случае от карты к вашему мозгу. Эверетт вычислил, что в точности происходит с волновой функцией: она превращается в описание суперпозиции двух различных конфигураций частиц, составляющих вас и карту. Три ключевых момента:

  1. Эксперимент переводит ваше сознание в два состояния сразу. Это, по сути, нелетальная версия эксперимента с котом Шредингера – с вами в роли кота.

  2. Эти два состояния сознания совершенно ничего не знают друг о друге.

  3. Состояние вашего сознания оказывается связанным с состоянием карты таким образом, что все кажется согласованным. (Волновая функция не описывает никаких конфигураций частиц, при которых вы воспринимали бы карту лежащей лицом верх, когда она лежит лицом вниз)

Итак, Эверетт понял: несмотря на то, что существует лишь одна волновая функция и одна квантовая реальность (в которой множество частиц, составляющих нашу Вселенную, находится в двух местах одновременно), на практике это эквивалентно тому, что наша Вселенная расщепляется на две параллельных. В конце этого эксперимента будут существовать две ваши копии, и каждая субъективно ощущает себя реальной, но совершенно не воспринимает существование другой.

На этом месте голова должна по-настоящему закружиться. Расщепление параллельных вселенных происходит постоянно, делая число квантовых параллельных вселенных поистине ошеломляющим. Поскольку такое расщепление идет с момента нашего Большого взрыва, практически любая версия истории, которую можно вообразить, реально разыгрывается в одной из квантовых параллельных вселенных, если только она не нарушает физических законов. Короче говоря, Эверетт показал, что если волновая функция никогда не коллапсирует, то знакомая нам реальность – лишь самая вершина онтологического айсберга, ничтожно малая часть истинной квантовой реальности.

Автор книги до этого момента описал и другие виды мультивселенных. Одна из них основана на теории инфляции (про нее можно почитать в одном из прошлых моих конспектов книги «Скрытая реальность: Параллельные миры и глубинные законы Космоса». Поэтому небольшое отступление: Под нашей Вселенной мы подразумеваем физическую область пространства, свету из которой хватило времени, чтобы дойти до нас за 14 млрд лет, прошедших с момента нашего Большого взрыва, со всеми ее классическими наблюдаемыми свойствами. Другие такие сферические области, находящиеся далеко от нас в огромном или бесконечном пространстве Тегмарк назвал вселенными I или II уровня в зависимости от того, действуют ли там те же законы физики, что у нас.

Назовем квантовые параллельные вселенные, открытые Эвереттом, параллельными вселенными III уровня, а их совокупность – мультиверсом III уровня. Если вселенные I и II уровней находятся в старом добром трехмерном пространстве очень далеко, то вселенные III уровня могут в смысле нашего трехмерного пространства располагаться прямо здесь, но они отделены от нас в гильбертовом пространстве – абстрактном математическом пространстве с бесконечным числом измерений, в котором обитает волновая функция.

Волновая функция соответствует одной точке в этом бесконечномерном пространстве, а из уравнения Шредингера вытекает, что эта точка будет двигаться вокруг центра пространства на фиксированном расстоянии.

Позже эвереттовскую теорию назвали многомировой интерпретацией, и это название вошло в научный обиход. Однажды у Эверетта спросили, что математика теории прекрасна, но сильно беспокоит то, что мы не ощущаем постоянного расщепления параллельных версий самого себя. Эверетт ответил вопросом на вопрос: «А вы чувствуете, что вращаетесь вокруг Солнца со скоростью 30 км/с?». Иногда бывает трудно примирить то, во что мы верим, с тем, что мы ощущаем.

Иллюзия случайности

Известно, что, если повторять квантовый эксперимент много раз, обычно получаются разные результаты, которые кажутся случайными. Например, можно измерять направление спина множества одинаковым образом приготовленных атомов так, что получится внешне беспорядочная последовательность результатов. Квантовая механика не предсказывает эти исходы, а говорит лишь о вероятности каждого из них. Но эти вероятностные дела завязаны на постулате копенгагенской интерпретации о коллапсе, от которого Эверетт отказался.

По мере того, как число повторений эксперимента растет, ситуация становится все интереснее. Согласно теореме французского математика Эмиля Бореля, доказанной в 1909 году, если повторить эксперимент с картами бесконечно много раз, вы будете наблюдать выигрыш в 50 % проб почти во всех случаях. Поэтому в окончательной суперпозиции почти все ваши копии будут считать, что законы вероятности действуют, невзирая на то, что в стоящей за ними физике (уравнении Шредингера) нет никакой случайности.

Однако фокус с иллюзией случайности вовсе не специфичен для квантовой механики. Допустим, некая технология будущего позволила клонировать вас во сне и две ваши копии помещены в разные комнаты. Когда они проснутся, они будут ощущать, что номер на двери их комнаты совершенно непредсказуем и случаен. Если вы многократно повторите эксперимент по клонированию и запишете найденные номера комнат, то почти во всех случаях вы увидите, что зафиксированная последовательность нулей и единиц выглядит случайной, и нуль встречается в номере примерно в половине случаев.

Рис. 2.
Рис. 2.

Иными словами, обычная физика будет порождать иллюзию случайности (с вашей, субъективной точки зрения) в любой ситуации, когда вас клонируют. Фундаментальная причина того, что квантовая механика кажется случайной несмотря на то, что волновая функция эволюционирует детерминистически, состоит в том, что, согласно уравнению Шредингера, волновая функция с единственным вашим экземпляром может эволюционировать в такую, согласно которой ваши клоны существуют в параллельных вселенных.

Работа Хью Эверетта все еще остается спорной, но, Тегмарк думает, он все-таки был прав и волновая функция никогда не коллапсирует. Тегмарк считает, что однажды его признают гением, равным Ньютону и Эйнштейну – по крайней мере, в большинстве параллельных вселенных. К сожалению, в нашей Вселенной его теорию десятилетиями игнорировали. Специальную теорию относительности тоже встретили с подозрением (особенно учитывая тот факт, что ее выдвинул не представитель академического сообщества, а служащий патентного бюро), но ее нельзя было игнорировать, поскольку Эйнштейн уже сделал себе имя другими открытиями.

Диссертация Эверетта оставляла открытым важный вопрос: если крупный объект может находиться в двух местах одновременно, почему мы этого никогда не наблюдаем? Конечно, если вы измерите его положение, две ваши копии в двух возникших параллельных вселенных обнаружат его каждая в определенном месте. Но такой ответ оказывается недостаточным: тщательные эксперименты свидетельствуют, что крупные объекты никогда не ведут себя так, как если бы они находились в двух местах сразу, даже если вы на них не смотрите. В частности, они никогда не проявляют волноподобных свойств, которые порождает квантовый интерференционный узор.

Квантовая цензура

Коллапс волновой функции можно элегантно описать посредством числовых таблиц, на квантово-физическом языке называемых матрицами плотности. В них закодировано не только состояние чего-либо (то есть волновая функция), но и, возможно, наше неполное знание этой волновой функции. Так, если нечто может находиться лишь в двух местах, то знание этого можно описать таблицей чисел размером два на два:

Матрицы плотности – это обобщения волновых функций. Каждой волновой функции соответствует матрица плотности, а каждой матрице плотности – уравнение Шредингера. Волновую функцию Ψ можно представлять, как комплексное число Ψi, заданное для любого возможного состояния i, и тогда соответствующая матрица плотности будет Pij = ΨiΨj*, где звездочка означает комплексное сопряжение. Если вы не знаете волновой функции объекта, а известна только вероятность того, что он имеет определенную волновую функцию, то надо использовать матрицу плотности, которая представляет собой взвешенное среднее от матриц плотности соответствующих этим волновым функциям.

В обоих случаях вероятность того, что мы найдем его в каждом из мест, составляет 0,5. Это кодируется двумя числами на диагонали обеих матриц. Остальные два числа в каждой таблице (недиагональные элементы матрицы плотности) кодируют разницу между квантовой и классической неопределенностями. В случае, когда они тоже равны 0,5, мы имеем дело с квантовой суперпозицией (кот Шредингера либо жив, либо мертв), но, когда они равны нулю, фактически все сводится к старой доброй классической неопределенности. Так что если вы сможете заменить недиагональные элементы нулями, то превратите «и» в «или» и вызовете коллапс волновой функции.

Согласно копенгагенской интерпретации, наблюдатель некоторым образом обнуляет эти недиагональные члены. Если имеется изолированная система, которая не взаимодействует больше ни с чем, то, применяя уравнение Шредингера, легко доказать, что эти нежелательные числа никогда не исчезнут. Но реальные системы почти никогда не бывают изолированными.

Тегмарк пришел к выводу, что квантовая механика требует секретности: объект может быть найден в двух местах сразу в состоянии квантовой суперпозиции лишь до тех пор, пока его положение остается в секрете для всего остального мира. Если секрет раскрывается, все эффекты квантовой суперпозиции становятся ненаблюдаемыми, и во всех практических отношениях он находится либо здесь, либо там, а вы просто не знаете, где. Даже если один единственный фотон отразится от объекта, информация о его местоположении также утечет: она будет закодирована в последующем положении фотона.

Нужен ли для коллапса волновой функции наблюдатель-человек? Теперь можно сказать, что сознание никакой роли не играет, поскольку и одна-единственная частица может осуществить этот трюк: одиночный фотон, отразившись от объекта, вызовет тот же эффект, что и наблюдающий его человек. Квантовое наблюдение связано не с сознанием, а просто с передачей информации. Вот почему мы никогда не видим макроскопические объекты в двух местах сразу, даже если они действительно находятся одновременно в двух местах: не потому, что они большие, а потому что их трудно изолировать. Мы по определению не можем увидеть что-либо, не ударив по нему фотоном, поскольку видеть предмет можно только благодаря отраженным им фотонам (свету).

Напротив, если откачать молекулы воздуха хорошим вакуумным насосом, то электрон, как правило, сможет просуществовать около секунды без столкновений с чем-либо, и этого времени более чем достаточно, чтобы продемонстрировать странные свойства квантовой суперпозиции. Так, электрон затрачивает в квадриллион раз меньше времени (около 10–15 секунды) на один оборот внутри атома, а значит, ничто не помешает ему находиться со всех сторон атома сразу. По сути хотя фотон в основном разрушает квантовую суперпозицию, части ее позволяет уцелеть: суперпозиция остается сравнимой по ширине с длиной волны фотона. Фотон, имеющий длину волны 0,0005 мм, действует практически так же, как наблюдатель, способный измерить положение предмета с точностью до 0,0005 мм. Все частицы ведут себя, как волны, и характеризуются длиной волны. Когда любая частица отскакивает от чего-либо, квантовая суперпозиция в масштабах, превосходящих длину ее волны, разрушается.

Рис. 3.
Рис. 3.

Почему ваш мозг – не квантовый компьютер

Пенроуз и другие выдвинули шокирующее предположение: возможно, у вас уже есть квантовый компьютер – в голове! Они предположили, что наши мозги являются квантовыми компьютерами и что это ключевой момент для понимания природы сознания.

Нейроны головного мозга передают электрические сигналы, перемещая атомы натрия и калия, у каждого из которых не хватает электрона (а потому они несут положительный электрический заряд). Если подключить отдыхающий нейрон к вольтметру, тот определит, что напряжение между внутренней и наружной областями клетки составляет 0,07 В. Если одно из окончаний нейрона снизит это напряжение, в клеточной мембране откроются чувствительные к напряжению каналы, заряженные атомы натрия начнут проходить по ним, напряжение снизится еще сильнее, и поток атомов усилится. Это цепная реакция, называемая разрядом, распространяется по всей длине нейрона со скоростью до 300 км/ч, пропуская внутрь клетки около миллиона атомов натрия. Аксон вскоре восстанавливается, и быстрые нейроны могут повторять этот процесс разряда более тысячи раз в секунду.

Рис. 4.
Рис. 4.

На рис. 4 схематическое изображение нейрона (слева), участка его длинного отростка, называемого аксоном (посередине) и фрагмента мембраны аксона (справа). Большая доля площади аксона покрыта непроводящим веществом миелином, но на нем есть небольшие оголенные участки (примерно каждые полмиллиметра), где концентрируются чувствительные к электрическому напряжению натриевые и калиевые каналы. Когда нейрон находится в суперпозиции состояний возбуждения и покоя, около 1 млн атомов натрия (Na) находится в суперпозиции состояний внутри и снаружи клетки (справа)

Теперь предположим, что мозг – действительно квантовый компьютер и разряд нейронов каким-либо образом вовлечен в эти вычисления. Тогда отдельный нейрон должен быть способен находиться в суперпозиции выдавшего и не выдавшего разряд, а значит, около миллиона атомов натрия должны находиться в двух местах одновременно – внутри и снаружи нейрона. Квантовый компьютер работает лишь постольку, поскольку его состояние остается тайной для мира. Так долго ли нейрон может хранить в секрете, выдал он разряд или нет? Вычисления Тегмарка показали ему, что «очень недолго»: около десяти миллиардных долей триллионной доли секунды (10–20 с). Он также обсчитал другую модель Роджера Пенроуза, в которой квантовые вычисления выполняются не нейронами, а микротубулами, элементами цитоскелета клеток, и обнаружил, что они поддаются декогеренции примерно за 10–13 секунды. Чтобы наши мысли соответствовали квантовым вычислениям, они должны завершаться прежде, чем случится декогеренция, так что нам следует думать со скоростью 10 000 000 000 000 мыслей в секунду.

На самом деле не удивительно, что мозг не работает подобно квантовому компьютеру. Ученые, которые пытаются построить квантовый компьютер, ведут затяжную войну с декогеренцией и обычно изолируют свои устройства в холодном темном вакууме, чтобы сохранить их состояние в секрете от остального мира, в то время как мозг – теплое влажное место, отделы которого не изолированы.

Субъект, объект и среда

Фейнман подчеркивал, что квантовая механика разделяет нашу Вселенную на две части: рассматриваемый объект и все остальное (среда). Но Тегмарк чувствовал, что в этой квантовой головоломке отсутствует важный элемент – сознание. Как показано в работе Эверетта, понимание процесса наблюдения требует подключения третьей части Вселенной – состояния сознания наблюдателя, отмеченного словом «субъект» на рис. 5.

Здесь имеется в виду лишь те отделы мозга, которые отвечают за сознательное восприятие

Рис. 5.
Рис. 5.

Удобно разбивать мир на три части: соответствующую вашему субъективному восприятию (субъект), ту, которая подвергается изучению (объект), и все остальное (среда). Взаимодействие между тремя частями вызывает качественно различные эффекты, благодаря которым складывается единая картина, включающая и декогеренцию, и кажущийся коллапс волновой функции.

В конце концов, разговор о наблюдениях без упоминания сознания напоминает дискуссию о близорукости без упоминания глаза. Тегмарк думает, это объясняется тем, что поскольку мы не понимаем, как устроено сознание, большинство физиков чувствует себя некомфортно, даже упоминая его в разговоре, из опасения, что их сочтут излишне философствующими. Однако одно лишь непонимание чего-либо вовсе не означает, что мы должны это игнорировать, надеясь притом получить корректные ответы. Следует предположить, что наше сознание возникает в результате удивительно сложного движения частиц, составляющих мозг, и эти частицы подчиняются уравнению Шредингера, подобно всем остальным частицам.

Тегмарк сделал следующе выводы на основе своей статьи (https://arxiv.org/pdf/gr-qc/9310032.pdf). Стандартные учебники концентрируются лишь на той части уравнения Шредингера, которая управляет объектом (например, атомом), следуя духу редукционизма (согласно которому такие вещи должны анализироваться сами по себе, без учета целого). Взаимодействие между субъектом и объектом дает параллельные вселенные Эверетта, распространяя квантовые суперпозиции с объекта на вас, субъект. Взаимодействие между средой и объектом дает декогеренцию, объясняя, почему крупные объекты никогда не показывают признаков странного квантового поведения вроде нахождения в двух местах одновременно. На практике попытки избавиться от декогеренции безнадежны в обычных условиях, но даже в мысленном эксперименте (скажем, при повторении эксперимента с квантовыми картами в темном холодном помещении без воздуха и с единственным фотоном, падающим на карту и затем наблюдаемым вами) существенного отличия не будет. Если карта находится в двух местах сразу, то же самое должно быть и с фотоном. Значит, по крайней мере один нейрон зрительных нервов перейдет в состояние суперпозиции выдавшего и не выдавшего разряд в тот момент, когда вы взглянули на карту. Эта суперпозиция испытает декогеренцию примерно через 10–20 секунды.

Декогеренция все же не до конца объясняет, почему вы никогда не ощущаете странностей квантового мира, поскольку мыслительный процесс (внутренняя динамика субъекта) может порождать странные суперпозиции знакомых состояний сознания. Однако, на помощь приходит третье из представленных на рис. 5 взаимодействий – между субъектом и средой. Тот факт, что нейроны испытывают декогеренцию значительно быстрее, чем обрабатывают информацию, означает, что если сложная схема разрядов нейронов в мозге имеет какое-либо отношение к сознанию, то декогеренция мозга будет препятствовать переживанию странных суперпозиций.

Взаимодействие субъекта и среды помогает связать и другие концы. Войцех Зурек показал, что декогеренция делает для нас еще одно важное дело. Она не только объясняет, почему крупные объекты никогда не видимы в двух местах сразу, но и дает понимание, в чем особенность обычных состояний (таких как пребывание лишь в одном месте). Из всех состояний, которые квантовая механика допускает для крупных объектов, эти обычные состояния являются одними из самых устойчивых к декогеренции, и именно они выживают.

Итак, мы не знаем, как устроено наше сознание, но знаем наверняка, что вся информация, поступающая в сознание из окружающего мира, должна сначала пройти по нейронам от органов чувств, например, от глаз по зрительным нервам или от ушей по слуховым нервам, а они испытывают декогеренцию за смехотворно короткое время. Так что к моменту, когда мы субъективно воспринимаем впечатление о внешнем мире, декогеренция уже завершилась, а это гарантирует невозможность восприятия нами квантовых странностей и объясняет, почему мы воспринимаем лишь устойчивые обычные состояния.

Как показано на рис 5, измерение и декогеренция отражают взаимодействие объекта соответственно с субъектом и со средой. Хотя процессы измерения и декогеренции могут выглядеть различно, энтропия открывает между ними интересный параллелизм: нехватка у нас информации об объекте является очень важной величиной, в физике называемой энтропией. Если объект ни с чем не взаимодействует, его энтропия остается постоянной: спустя секунду вы будете знать о его состоянии ровно столько же, сколько знаете сейчас, поскольку можете вычислить его будущее состояние по исходному с помощью уравнения Шредингера. Если объект взаимодействует с вами, то обычно вы получаете о нем больше информации, и его энтропия уменьшается. Но если объект взаимодействует со средой, вы обычно теряете информацию о нем, и его энтропия увеличивается. Короче говоря, Тегмарк представляет это так: энтропия объекта убывает, когда вы на него смотрите, и возрастает, когда не смотрите. Декогеренция – это просто измерение, результатов которого вы не знаете. Стремясь к большей строгости, можно точнее сформулировать второе начало термодинамики:

  1. Энтропия объекта не может убывать, если он не взаимодействует с
    субъектом.

  2. Энтропия объекта не может возрастать, если он не взаимодействует
    со средой.

Кому интересно и имеет хорошую подготовку в физике и математике, то Макс Тегмарк опубликовал по этому вопросу статью.

Квантовый суицид

Макс Тегмарк потратил массу времени, размышляя, возможен ли эксперимент, который убедил бы в том, что эвереттовские параллельные вселенные реальны, и, наконец, он придумал такой – квантовый суицид.

Оборудование представляло собой «квантовый пулемет», который стреляет в зависимости от исхода квантового измерения. То есть всякий раз, когда пулемет запускается, он помещает частицу в состояние суперпозиции двух равновероятных состояний (скажем, вращения по или против часовой стрелки), а затем выполняет измерение частицы. Если оказывается, что она находится в первом из двух состояний, пулемет выстреливает, а в противном случае лишь громко щелкает. Интервал времени между квантовым измерением и выстрелом гораздо короче характерного времени человеческого восприятия – скажем, 1/100 секунды.

Теперь предположим, что вы запускаете квантовый пулемет в автоматическом режиме, и он срабатывает один раз в секунду. Нетрудно предсказать, что вы услышите кажущуюся случайной последовательность выстрелов и щелчков. И тут вы делаете нечто экстремальное: помещаете голову на линию огня и ждете. Что вы ожидаете почувствовать? Это зависит от того, реальны эвереттовские параллельные вселенные или нет. Если нет, то у каждого квантового измерения только один исход, так что секунду спустя вы определенно будете мертвы либо живы. Вероятность прожить n секунд составляет 1/2n, так что ваши шансы прожить минуту составляют менее 10–18. С другой стороны, если эвереттовские параллельные вселенные реальны, то через секунду будут существовать две параллельные вселенные: в одной вы живы, а во второй мертвы. Иными словами, существует ровно одна ваша копия, обладающая восприятием и до, и после события-триггера, а поскольку процесс протекает слишком быстро для восприятия, предсказание состоит в том, что вы услышите щелчок со стопроцентной вероятностью. Подождите немного, и вы обнаружите нечто странное: поместив голову на линию огня с, казалось бы, случайной чередой выстрелов и щелчков, вы услышите щелчок-щелчок-щелчок и т. д. После десяти щелчков вы сможете исключить коллапс волновой функции с уверенностью 99,9 % в том смысле, что если коллапс волновой функции действительно происходит, то вероятность быть мертвым к этому моменту превысит 99,9 %. Через минуту вероятность того, что Эверетт ошибался, будет составлять 10-18. Чтобы снять любые сомнения в исправности квантового пулемета, вы убираете голову с линии огня и обнаруживаете, что выстрелы и щелчки снова начали чередоваться.

Подобным образом размышлял также австрийский математик Ханс Моравек в книге об искусственном интеллекте «Дети разума» (1988)

Но что если реален мультиверс I уровня? Тогда вначале должно существовать бесконечно много параллельных вселенных, которые содержат вас в субъективно неразличимых состояниях сознания, но с неизмеримо малыми различиями в начале эксперимента. Через секунду вы будете мертвы в половине этих вселенных, но независимо от того, сколько раз эксперимент повторяется, всегда найдутся такие вселенные, где вас никогда не застрелят. Иными словами, описанный жуткий эксперимент может раскрыть существование не только параллельных вселенных III (квантового) уровня, но и параллельных вселенных в более широком смысле.

Мультиверс объединенный

Не представляют ли мультиверсы I и III уровней в некотором смысле одно и то же? Можно ли их каким-либо образом объединить, подобно тому, как Максвелл объединил электричество и магнетизм в электромагнетизм, а Эйнштейн объединил пространство и время в пространство-время? С одной стороны, кажется, что их природа различна. Параллельные вселенные I уровня находятся где-то очень далеко в нашем старом добром трехмерном пространстве, а параллельные вселенные III уровня могут располагаться прямо здесь, в смысле наших трех измерений, но они отделены от нас в гильбертовом пространстве (абстрактном пространстве с бесконечным числом измерений, в котором обитает волновая функция). С другой стороны, мультиверсы I и III уровней имеют много общего. Жауме Гаррига и Александр Виленкин показали, что параллельные вселенные I уровня, порожденные космологической инфляцией, содержат те же последовательности событий, что и эвереттовские квантовые параллельные вселенные. Если квантовое событие вызывает два события, происходящих в квантовой суперпозиции, фактически расщепляя ваше будущее на две параллельные квантовые ветви, то параллельный квантовый исход, о котором вы сейчас ничего не знаете, происходит также и здесь, в вашей квантовой ветви, но очень далеко в пространстве. И был еще один источник беспокойства: Энтони Агирре. «Что, если одни параллельные вселенные равнее других?» – спрашивал он.

Рис. 6 - Сравнение параллельных вселенных уровней I и III.
Рис. 6 - Сравнение параллельных вселенных уровней I и III.

Энтони обращал внимание на то, что объяснение квантовых вероятностей, которое дал Тегмарк, отлично подходит, если исходы имеют одинаковую вероятность, но все усложняется, если вероятности неодинаковы. Так Тегмарка часто спрашивали, верит ли он в реальность параллельных вселенных Эверетта. Ответ «Да, но… хм… как бы сказать… Некоторые из них реальнее, чем другие» звучит крайне неубедительно.

Однако в итоге Тегмарк понял, как разом решить обе проблемы: объединить два уровня мультиверсов и объяснить неравные вероятности. Рис. 7 иллюстрирует ключевую идею.

Рис. 7.
Рис. 7.

Допустим, вы собираетесь выполнить эксперимент с квантовыми картами, слегка наклонив карту, так что ожидаете увидеть ее упавшей лицом вверх с вероятностью 2/3. В традиционном виде (слева в каждом прямоугольнике на рис. 7) вначале имеется одна ваша копия, а затем, после эксперимента, – одна либо две копии, в зависимости от того, коллапсирует волновая функция или нет.

Пусть теперь существует мультиверс I уровня, как и предполагает современная космология. Это означает, что бесконечное число неотличимых от вас копий выполняет точно тот же эксперимент на других планетах очень-очень далеко (на рисунке это показано рядом нейтральных смайликов).

Что в итоге? Если волновая функция коллапсирует, получится один случайный исход для всего бесконечного пространства (мультиверса I уровня), так что вы окажетесь довольным на 2/3 планет и опечаленным на 1/3. Если же Эверетт прав в том, что коллапса не происходит, то результатом станет целое бесконечное пространство в квантовой суперпозиции различных состояний, в каждом из которых вы радуетесь на одних планетах и печалитесь на других. Все эти состояния пространства оказываются неотличимы друг от друга: вы счастливы ровно на 2/3 бесконечного множества планет. Любая конечная последовательность планет со счастливыми и несчастливыми исходами в одном из этих состояний обнаружится где-то в ином месте пространства в каждом из остальных состояний. Можно подумать, что должны существовать отличающиеся состояния пространства, скажем, такое, где вы счастливы на всех планетах. Однако, опираясь на уравнение Шредингера и математические свойства гильбертова пространства, Тегмарк смог доказать, что получаемая в итоге волновая функция равна простой суперпозиции бесконечного числа неотличимых состояний.

Великая дискуссия о том, коллапсирует ли волновая функция, завершилась великим разочарованием: это попросту не имеет значения. На рис. 7 показано, что независимо от того, прав Эверетт или нет, вы будете счастливы на 2/3 планет. Параллельные вселенные III уровня (квантовые) можно спокойно игнорировать, поскольку все они неразличимы. В этом смысле мультиверсы уровней I и III объединяются: если имеется бесконечное пространство с мультиверсом I уровня, то можно игнорировать связанные с ним параллельные вселенные III уровня, поскольку на практике все они представляют собой лишь идентичные копии. Не исключено, что III уровень можно объединить и со вторым, но пока Тегмарк с коллегами не смогли это доказать.

Рис. 7 демонстрирует происхождение неравных вероятностей, перенося множественные миры Эверетта в наше трехмерное пространство. Различные исходы случаются не только где-то в труднопредставимом математическом гильбертовом пространстве, но и где-то очень далеко в нашем собственном пространстве. Суть в том, что после того, как карта упала, но до того момента, когда вы открыли глаза, у вас нет способа узнать, какой из множества копий самого себя вы являетесь, поскольку до этого последнего момента все они чувствуют себя субъективно неразличимыми. Поскольку вы знаете, что 2/3 ваших копий, открывая глаза, видят карту лежащей лицом вверх, вам кажется, что вы случайным образом обнаружите ее в этом положении с вероятностью 2/3.

Это позволяет предложить космологическую интерпретацию квантовой механики. В ней мы интерпретируем волновую функцию объекта не как описание воображаемой совокупности возможностей для данного объекта, а как реальную пространственную совокупность идентичных копий объекта, существующих в бесконечном пространстве. Более того, испытываемая вами квантовая неопределенность просто отражает вашу неспособность определить свое положение в мультиверсе I уровня, то есть узнать, какая из бесконечного числа ваших копий, разбросанных по космосу, обеспечивает ваше субъективное восприятие.

Кому интересно и имеет хорошую подготовку в физике и математике, то Макс Тегмарк в соавторстве с Энтони опубликовали по этому вопросу статью.

Много миров – или слов?

Так что же делать с квантовыми соображениями? Можно подумать, что если эксперты продолжают спорить об этом на протяжении столетия после открытия квантовой механики, то они, вероятно, проспорят и следующее столетие. Однако за это время контекст дискуссии существенно изменился.

Во-первых, из теоретических открытий Эверетта и других стало понятно, что даже если отбросить спорный постулат о коллапсе волновой функции, сохранив лишь голый скелет квантовой механики (в которой всегда выполняется уравнение Шредингера), мы по-прежнему будем субъективно ощущать, будто волновая функция, подчиняясь всем законам вероятности, коллапсирует, когда мы производим наблюдения, и оставляет нас в счастливом неведении относительно любых квантовых параллельных вселенных.

Во-вторых, космологические открытия предполагают, что мы все равно оказываемся в параллельных вселенных, даже если Эверетт ошибался. Более того, эти параллельные вселенные I уровня сливаются с квантовыми.

В-третьих, поддержка той идеи, что квантовая гравитация неким образом приводит к коллапсу волновой функции, сама коллапсировала из-за прорыва в теории струн, известного как AdS/CFT-дуальность. Главный смысл: найдено математическое преобразование, показывающее, что определенные квантовые теории поля с гравитацией могут быть реинтерпретированы как другие квантовые теории поля без гравитации. Очевидно, что гравитация не вызывает коллапса волновой функции, раз само ее присутствие – в значительной мере вопрос интерпретации.

В-четвертых, все более точные эксперименты исключили множество попыток поверхностного объяснения квантовых странностей. Например, нельзя ли заменить кажущуюся квантовую случайность некими неизвестными величинами, содержащимися внутри частиц (скрытыми переменными), которые можно измерить в определенных сложных экспериментах? Спустя много лет технологии наконец достигли уровня, позволившего осуществить эти эксперименты, и в результате возможность существования скрытых переменных была исключена.

Не может ли быть так, что существуют не открытые пока поправки к уравнению Шредингера, которые приводят к распаду квантовых суперпозиций для достаточно крупных объектов? Простой эксперимент с двухщелевой интерференцией был успешно повторен с объектами, превосходящими по размерам отдельные элементарные частицы: атомами, небольшими молекулами и даже с молекулами фуллерена из 60 атомов углерода.

Резюме

  • В квантовой теории существует нечто более фундаментальное, чем наше трехмерное пространство с частицами в нем, – волновая функция и бесконечномерное вместилище (гильбертово пространство), в котором она обитает.

  • Согласно этой теории, частицы могут создаваться и уничтожаться, а также находиться в нескольких местах сразу, но есть, была и всегда будет лишь одна волновая функция, движущаяся по гильбертову пространству так, как предписывает ей уравнение Шредингера.

  • Эта квантовая теория (в которой уравнение Шредингера всегда выполняется) предсказывает существование параллельных вселенных.

  • Из нее также вытекает, что квантовая случайность – это иллюзия, вызванная квантовым клонированием вас самих.

  • Эта теория также предсказывает эффект цензуры, называемый декогеренцией. Она скрывает от нас большинство таких причуд, порождая впечатление коллапса волновой функции.

  • Декогеренция в нашем мозге происходит постоянно, что разрушает популярные предположения о «квантовом сознании».

  • Квантовый мультиверс объединяется с пространственным мультиверсом, так что волновая функция системы описывает ее бесконечные копии, разбросанные по пространству, а квантовая неопределенность отражает наше незнание того, какую именно копию мы наблюдаем.

  • Если мы живем в бесконечном однородном пространстве, как предполагается в стандартной космологической модели, то не имеет значения, коллапсирует ли в действительности волновая функция: все многочисленные эвереттовские миры неразличимы, и коллапс не препятствует тому, чтобы все квантовые исходы в действительности имели место.

  • Волновая функция и гильбертово пространство, которые составляют, по-видимому, самую фундаментальную физическую реальность, – чисто математические объекты.

Список статей Тегмарка, которые упоминались в этом конспекте:

  1. Apparent wave function collapse caused by scattering

  2. How unitary cosmology generalizes thermodynamics and solves the inflationary entropy problem

  3. Born in an Infinite Universe: a Cosmological Interpretation of Quantum Mechanics

Ссылки на все части

Комментарии (125)


  1. phanerozoi_evidence
    22.08.2021 11:19

    Интересно было почитать, что-то далеко от своей области. Мне понравилась. + в карму и в статью заслуженно


    1. ArkadiyXIII Автор
      22.08.2021 12:53
      +2

      Спасибо


  1. novoselov
    22.08.2021 11:46
    +6

    К многомировой теории есть несколько вопросов:

    1. Если каждый квантовый процесс вызывает расщепление вселенной, то существуют вселенные где любая монета падает только решкой вверх, где укус комара вызывает цепную термоядерную реакцию, где наблюдаемые законы физики меняются произвольным образом, например вы можете бросить камень прямо, но в одном случае он при падении будет скакать на одном месте пока не остановится, а в другом будет скакать по круга. При этом это никак не противоречит общим законам физики. Каков в таком случае процент стабильных вселенных?

    2. В разных вселенных поведение окружающих людей тоже может меняться произвольным образом, например есть вселенные где вы прочитав эти строки без видимой причины выйдете в окно. Опять же это не противоречит никаким законам физики, просто так карта легла. В таких условиях нельзя говорить не только о свободе выбора, но и о стабильных причинно-следственных связях. Как предсказать разультат каких-то действий, если один случайный процесс может изменить исход?

    3. С учетом бесконечного множества комбинаций различных явлений вокруг нас кто-то уже должен был зафиксировать подобные отклонения, а в эпоху интернета и моментального распространения информации это стало бы известно очень быстро. Почему никто не наблюдает таких странностей в физических явлениях или поведении людей?

    4. Как результат не объяснется непрерывность и единственность сознания. Даже если мы не ощущаем расщепления, объяснение "мы просто находимся не в той версии вселенной" не прокатит, для большинства расщепленных сознаний статистически должны наблюдаться странные явления. Можно например просидеть осень, зиму, весну и почти все лето под одной конкретной яблоней ожидая когда на вас упадет яблоко и ничего не произойдет до "того самого момента", но в масштабах земли есть миллионы яблок которые падают непрерывно несколько месяцев подряд. Если вы не видели как падают яблоки есть миллионы других людей которые расскажут вам об этом. В рамках вашего расщепленного сознания чужое знание и ваш собственный опыт равносильны, источник информации неважен. Но вопреки этому наше сознание всегда остается в нужной версии вселенной. Как это объяснить?


    1. ArkadiyXIII Автор
      22.08.2021 12:43

      Спасибо, что выделили время на мой конспект. Вопросы хорошие и объемные, но к сожалению, сейчас недостаточно времени, чтобы нормально их проанализировать и ответить в нужной мере. На данный момент у меня пока следующие аргументы:

      По поводу 2. В статье упоминалось во время описания мысленного эксперимента «квантовые карты», что волновая функция не описывает никаких конфигураций частиц, при которых вы воспринимали бы карту лежащей лицом верх, когда она лежит лицом вниз, то есть все должно быть согласованно. Если нет возможности предсказать что-то, так это только из-за того, что недостаточно информации, чтобы это сделать. Для 100% предсказания нужно знать текущее состояние всех элементов, которые составляют эту систему, то есть в нашем случае состояние всех элементарных частиц из наблюдаемой вселенной как минимум (если извне, то что мы не наблюдаем не влияет на нашу наблюдаемую часть вселенной). Есть мысленный эксперимент Демон Лапласа — вымышленное разумное существо, способное, восприняв в любой данный момент времени положение и скорость каждой частицы во Вселенной, узнавать её эволюцию как в будущем, так и в прошлом. Лаплас придумал это существо для наглядной демонстрации степени нашей неосведомленности и необходимости в статистическом описании некоторых реальных процессов в окружающем мире.

      Из этого вытекает, что не может быть никаких странностей (ваш 3-4 пункты). На все можно найти свою причину.


      1. kokonT
        22.08.2021 20:43

        Аркадий! Прошу пояснить, откуда берётся материя (энергия) для создания всех тех вселенных, которые возникают в каждом акте квантовой интерференции.



    1. SergioShpadi
      22.08.2021 12:53
      +7

      1. Да, будут существовать Вселенные, в которых будут происходить маловероятные последовательности событий, но их количество будет минимально. Например в ходе эксперимента с подкидыванием "квантовой монетки" из десяти попыток будет Вселенная, в которой десять раз выпадет решка, но такая Вселенная будет одна из тысячи двадцати четырёх. В таких мирах будет невозможно установить реальный статистический закон, действующий во всей мультивселенной и легко обнаруживаемый в большинстве миров. Но ни в одном из таким миров не будут нарушаться законы физики, поэтому камень всегда будет падать вниз.

      2. Законы причинно-следственных связей на глобальном уровне не нарушится. Так как любые случайные квантовые события на глобальном уровне не проявляются. Они "нейтрализуются", когда из хаоса микромира возникает порядок макромира.

      3. Отклонения от чего? Вы проводите квантовое измерение: из 1000 попыток вам 502 раза выпало один результат, а 498 раз другой. Почему от этого люди должны сходить с ума и выпрыгивать из окон, а камни взлетать в небо?

      4. Сознание также расщепляется: в одной параллельной Вселенной вы видите один результат, в другой - другой. И что вообще значит "в нужной версии вселенной"? Нужной кому?


    1. 0serg
      22.08.2021 13:00
      +4

      Разные вселенные не «равновероятны». Вселенные где нарушаются то что мы называем законами физики существуют, но их очень мало. Грубо говоря чтобы оставаться в такой вселенной Вам надо 10^20 раз в секунду кидать честную монетку и каждый раз выкидывать «решку». Соответственно вероятность для Вашего сознания оказаться именно в такой вселенной а не «нормальной» которых на многие порядки больше для любых практических целей равна 0.


    1. IgorDev
      22.08.2021 13:35

      Почему не наблюдают? Вы не слышали о падающих кирпичах на голову? Не наблюдает большинство, а для некоторых наша нормальная вселенная оказывается именно такой непредсказуемой, как Вы описали. Кто то выйдет в окно прочитав вот эти строки... кто то другие


    1. Rating-100
      22.08.2021 14:28

      А у меня только один вопрос:

      Откуда энергия, чтобы плодить бесконечное количество новых вселенных каждую секунду?


      1. iShrimp
        22.08.2021 16:19

        А почему нас должно это заботить? Если наша вселенная вычисляется на компьютере, то мир, в котором существует этот компьютер, может быть сколь угодно сложным (его размерность может превышать число всех возможных состояний нашей вселенной).


        1. dcoder_mm
          22.08.2021 23:07
          +4

          Если наша вселенная вычисляется на компьютере

          А она должна вычисляться на компьютере только потому, что мы придумали компьютеры и восхищаемся тем, как с их помощью можно моделировать сложные процессы, да?

          А когда компьютеров еще не было, думали что мир это кем-то написанная книга. Ведь книга может описать так много вещей.

          А до этого, что мир это чей-то сон. Ведь во сне можно увидеть целый новый мир.


          1. iShrimp
            23.08.2021 19:54

            Не привязывайтесь к слову "компьютер", это просто общеизвестный пример системы, способной удерживать в памяти и симулировать некий виртуальный (вымышленный) мир в соответствии с некими правилами. Если бы мы не жили в эпоху компьютеров, а моделировали миры в своём воображении или в химических средах, то и слово было бы другим, суть не в этом.

            Откуда энергия, чтобы плодить бесконечное количество новых вселенных каждую секунду?

            Суть в том, что генерация новых вселенных (в понимании Эверетта) не требует затрат какой-либо энергии, которая может "кончиться", и нас не должно удивлять их огромное количество. Согласно его теории, это естественный процесс, лежащий в основе эволюции нашего мира.

            Если верна гипотеза симуляции, то можно считать, что новые варианты Вселенной постоянно автоматически генерируются и добавляются в общий список.

            Я упоминаю эту гипотезу только потому, что это доступный способ объяснить возможное устройство мира. Мы не знаем, и, наверное, никогда не узнаем, верна ли она, так как (1) мы не можем выглянуть за пределы песочницы и (2) если бы мы даже и узнали что-то о внешнем мире, то скорее всего, не смогли бы ничего понять. Если этот симулятор способен хранить и обрабатывать экспоненциально растущее (с точки зрения Эверетта) количество информации, то, по-видимому, он устроен сложнее, чем всё, что мы можем себе вообразить.


      1. 0serg
        22.08.2021 16:24
        +2

        Энергия никуда не девается. В статье же написано что идея с «созданием» новой вселенной как таковая неверна. Вселенная всего одна, но ее волновая функция получается суперпозицией двух волновых функций каждая из которых соответствовала бы отдельной версии Вселенной. Просто чисто математически это можно рассматривать как то что было две идентичных копии Вселенной которые в какой-то момент перестали быть идентичными и начали расходиться.


      1. Bronx
        23.08.2021 08:28
        +2

        Энергия в физике — величина не абсолютная, она всегда считается от некоторого базового уровня, который называется вакуумом. Для законов физики важны соотношения энергий. Например, если вдруг при расщеплении абсолютно все соотношения сохраняются (скажем, делятся на 2), то никто ничего не заметит, это просто глобальная замена измерительных линеек.


        Но, вообще говоря, энергия связана с частотой волновой функции. При расщеплении же волны на две, все частоты (а значит и энергетические соотношения) сохраняются, изменяются лишь амплитуды. Амплитуда же волновой функции соответствует плотности вероятности — и уменьшение амплитуды каждой дочерней волны соответствует тому, что вероятность оказаться в одной из них равна 1/2.


        После того, как дочерние волны расходятся, они больше не взаимодействуют друг с другом, даже если пройдут сквозь друг друга, в силу принципа суперпозиции. Ну, примерно как мелкие волны на поверхности бассейна живут параллельной жизнью каждая, спокойно разделяя один бассейн на всех и не замечая этого. Если в бассейн опускается палка и волны делятся, они не задаются вопросом "откуда энергия, чтобы плодить новые волны" :)


        Когда каждая из ваших копий оказывается связанной с одной из половин волновой функции, её новая уменьшенная амплитуда становится для копии "новой нормой", т.е. с точки зрения каждой копии тот факт, что она тут живёт, всегда является "сбывшимся фактом" с вероятностью 1, и все прочие вероятности для неё автоматически перенормируются, как в теореме Байеса.


    1. vvadzim
      22.08.2021 14:40
      +1

      Миллионы яблок на протяжении месяцев и миллиарды людей - это очень, очень мало, чтобы можно было наблюдать описываемыя вами события по выходу в окно и скаканию камней. Вероятность любого необычного явления настолько мала, что вероятность наблюдать его за время жизни земли хоть одним из людей просто мизерная. А вы в своих вопросах продаёте как факт, что эта вероятность существенная. В этом ошибка.


    1. Scinolim
      22.08.2021 16:24
      -1

      Тут надо понимать что квантовая механика это статистический подход к описанию явлений. Совсем не обязательно всё должно работать именно так, как производятся вычисления. Для конкретной частицы есть одна траектория, никаких расщеплений не требуется. Но когда частиц много, мы не можем измерить каждую, а только в среднем сказать какая вероятность что какая-то из частиц окажется в данном состоянии.


      1. 0serg
        22.08.2021 16:41
        +1

        Нет, в рамках квантовой механики как раз расщепление требуется для каждой отдельной частицы. Например радиоактивный распад частицы требует расщепления. Эксперименты с прохождением частицы через две щели требуют расщепления. В том и прикол что статистические модели не работают, есть даже отдельная теорема о том что это можно проверить (неравенства Белла) и ее неоднократные проверки показали что наша реальность подчиняется квантовым законам а не классическим.


        1. Scinolim
          23.08.2021 10:53

          Расщепление вселенной имелось в виду.


    1. MetromDouble
      22.08.2021 22:11

      del


    1. MetromDouble
      22.08.2021 22:15

      Как это объяснить?

      Объяснение простое, но неприятное. Люди хотят сохранить аксиому о свободе воли (а это именно аксиома, так как её наличие нельзя никак доказать с помощью эксперимента), даже ценой ввода в дискурс параллельных измерений.

      Если мы отказываемся от аксиомы о свободе воли, то самой простой интерпретацией квантмеха становится супердетерминизм, который никаких лишних сущностей не требует.

      Подробнее в вики - теорема о свободе воли


    1. Sergey_Cheban
      23.08.2021 00:09
      +3

      где наблюдаемые законы физики меняются произвольным образом

      Законы физики одинаковы во всех вселенных. Но при этом вполне могут существовать вселенные, в которых наблюдаются имеются самые странные статистические аномалии. Если верить ЦИК РФ, то наша вселенная как раз такая. Также не исключено, что примером странной статистической аномалии является появление и сохранение жизни и разума.


    1. Vkuvaev
      23.08.2021 00:43

      Отличные вопросы. По 3-му, Можно прикинуть следующее: ок вселенных бесконечно много, эти аномалии должны наблюдаться регулярно(туннельный эффект, например?). Но на макро уровне бесконечно мала вероятность найти экземпляр вселенной, где все, или значительный макрообъем, элементарных частиц входящих в некий макро объект испытают идентичную квантовую аномалию которую можно достоверно интерпретировать как доказательство существования мультивселенной.

      Кстати, возможно кто-то и наблюдал, только никто не поверит рассказу :)


    1. Tarakanator
      24.08.2021 10:45

      Случайно созданная вселенная пригодна для жизни. По вашему это недостаточная странность?

      А не наблюдают в большем количестве, потому как вероятность крайне мала.
      Я перефразирую: мы наблюдаем только те крайнемаловероятные события, которые не можем не наблюдать.
      Примеры:
      1)рождение вселенной
      2)квантовый пулемёт


  1. v1000
    22.08.2021 12:18
    +3

    гравитация не вызывает коллапса волновой функции, раз само ее присутствие – в значительной мере вопрос интерпретации.

    с гравитацией вообще одни загадки, с одной стороны самая доступная для понимания вообще без каких-либо приборов, с другой стороны самая слабая из всех и наименее точно измеренная с использованием самых точных приборов.


  1. WFF
    22.08.2021 14:45

    Не очень понятно, сколько вселенных возникнет в случае не целочисленной, а дробной вероятности (например, 1/3).


    1. Krypt
      22.08.2021 16:05
      +1

      Насколько я понимаю идею, это «аналоговый» процесс: размеры регионов волновой функции, соответствующие этим «вселенным» будет соотносится как 1 (событие произошло) к 2 (событие не произошло)


      1. WFF
        22.08.2021 16:38

        Ок, неплохо, но не означает ли это детерминизм подобного мира с самого начала "сотворения"? Ведь мы убрали случайность с точки зрения подобной мультивселенной?


        1. 0serg
          22.08.2021 16:45
          +5

          А в том и состоит идея что наш мир 1) полностью детерминирован но 2) в нем одновременно существует бесконечное количество существующих параллельно версий «сознания» каждая из которых «видит» только свою «версию» (подмножество) мира. С точки зрения совокупности всех наблюдателей мир детерминирован, но с точки зрения каждого отдельного наблюдателя видно лишь случайно выбранное подмножество этого мира и вот этот случайный выбор уже не детерминирован.


          1. WFF
            22.08.2021 16:55

            Я правильно понимаю, что в этом случае вопрос возникновения подобного мира остается открытым, так как случайности больше нет?


            1. red75prim
              23.08.2021 15:42

              Все интерпретации квантовой механики основываются на одних и тех-же формулах. Например, формулы состояния Хокинга-Хартла, описывающие один из возможных сценариев возникновения вселенной, останутся теми-же самыми.

              Но в многомировой интерпретации они означают, что вселеная как целое до сих пор находится в суперпозиции безвременного "ничто" и развивающейся вселенной с большим взрывом и всем прочим. Безвременное "ничто" некому наблюдать, так что наблюдается только существующая вселенная.


  1. redlynx
    22.08.2021 16:00
    +1

    мне очень понравился Квантовый пулемет!

    Можно ли его, когда-нибудь, реализовать "физически" и экспериментальным путем продемонстрировать "наглядно"?

    Я с удовольствием бы поучаствовал в нем, ведь судя по описанию, понадобится чья-то живая голова:)

    С этим пулеметом мы будем разъезжать по Арканзасу и показывать фокусы в передвижном шапито для развлечения местных реднеков и просто фермеров, которые, пока, не знакомы с работой Эверетта и будут все принимать за чистую монету!


    1. Krypt
      22.08.2021 16:13
      +3

      Насколько я понял принцип, живой наблюдатель будет видеть такой результат просто потому, что все остальные его версии умерли, а мы отслеживаем именно живого наблюдателя. Для него же, если голову под квантовый пулемёт засунул кто-то другой — он выживет с шансом в 10^-18, и это всё будет выглядеть как «классическая» вселенная.
      То есть в эксперименте нужно участвовать именно экспериментатору… С сомнительными шансами на успех (только 10^-18 часть абстрактных «я» экспериментатора в волновой функции выживет)


    1. iShrimp
      22.08.2021 16:31
      +2

      Этот парадокс квантового бессмертия можно считать следствием ошибки выжившего. Только вам в вашем мире будет казаться, что вы выжили после 10 выстрелов. Другие наблюдатели увидят то, что и должны увидеть - что с вероятностью 1023/1024 вы будете застрелены. Ради интереса можно посмотреть фильм "Престиж".


      1. SergioShpadi
        22.08.2021 17:25
        -1

        А почему 1023/1024, а не 10/11? Ведь каждый раз при выстреле одна из копий умирает, и во Вселенной умершей копии считать раздвоение уже бессмысленно.


        Пример на три выстрела:

        • A - B B - C C C C - D D D D D D D D - восемь вариантов, ведь 2^3=8при раздвоении Вселенной

        • A - B - C - D - а вот так будет выглядеть ряд, если одна из копий умирает, ведь "мертвая" Вселенная уже не поменяет стейт (с мёртвого на мёртвый), стейт может измениться только с живого на мёртвый


        1. mayorovp
          22.08.2021 17:29

          Потому что вероятности независимых событий умножаются и не зависят от числа вселенных.


    1. Eugney
      22.08.2021 18:09
      +2

      Даже если теория квантового бессмертия верна, то непонятно, как гарантировать смерть, потому что пулемёт может покалечить экспериментатора, оставив при этом в живых. То есть на практике, вместо ожидаемой последовательности щелчков произвольной длительности, через какое-то время последует выстрел, который отправит в кому, лишив возможности продолжать эксперимент.

      Даже если ассистент будет добивать таких подранков, то всегда есть вероятность того, что у него не хватит духа или что-то ему помешает.

      Но вообще, если квантовое бессмертие существует, то по идее каждый из нас однажды станет самым старым человеком на планете. И если это действительно произойдёт, то это будет косвенным доводом в пользу этой теории.


      1. iShrimp
        22.08.2021 19:27
        +1

        каждый из нас однажды станет самым старым человеком на планете

        В своей вселенной каждый человек бессмертен, и он постоянно видит, как другие умирают, а сам он живёт и живёт. Но для нас, сторонних наблюдателей, вероятность увидеть его бессмертным (попасть в один мир с ним) стремится к нулю.


        1. HappyLynx
          22.08.2021 23:31
          +1

          Вот только, если теория квантового бессмертия верна, то возникает два основных вопроса:

          1. Каким образом частное сознание "выбирает" конкретную версию вселенной, в которой оно продолжает существовать, среди бесконечного множества вселенных в мультивселенной, в которых оно продолжает существовать?

          2. Что есть "существование" сознание, и что есть "прекращение существования сознания"?

          Субъективно я могу дать, пусть и не претендующий на полноту, ответ только на первый вопрос:

          "Выбор" осуществляется не в пользу наиболее "комфортного" (вне зависимости от определения понятия "комфорта") состояния сознания, а случайным образом с учетом вероятности соответствующего исхода. Т.е., если на пальцах, если из всех вселенных, образующихся после точки, где существованию сознания угрожает опасность, в 2/5 вселенных сознание перестает существовать, в 2/5 продолжает существовать но, допустим, вы получаете травму, а в 1/5 продолжает существовать и травму вы не получаете, то, согласно теории квантового бессмертия, у конкретной копии вашего сознания 2/3 шанса оказаться во вселенной, где вы получили травму, и 1/3 шанса оказаться во вселенной, где травму вы не получили.

          Данный ответ логическим образом вытекает из того, что в моей жизни (в той версии вселенной в которой существует моё сознание) были ситуации где я, например, ломал руку. Логично предположить, что более комфортными для моего сознания были бы те варианты вселенной, где я, пусть и крайне маловероятно, почти чудом, но руку не ломаю, однако руку я ломал. Точно так же в старости, когда у вас будет шанс смертельного инсульта с вероятностью 20%, шанс остаться без инсульта с вероятностью 5%, и 75% шанс инсульта с потерей речи, по теории квантового бессмертия вы не попадете в первые 20%, но все равно, с гораздо большей вероятностью угодите в группу вселенных с потерей речи, а не останетесь без инсульта.

          И, с учетом вышеизложенного, мы можем, если не ответить на второй вопрос, то, по крайней мере, определить ограничения на возможный ответ, в частности, что "комфорт" существования не является обязательным условием для существования сознания.

          А значит, чем ближе мы будем к своей старости, тем большую долю от всех вселенных, где наше сознание продолжает существовать, будут получать те варианты вселенной, где оно будет продолжать существовать, но во все менее и менее комфортных условиях. Инсульт, и вот вы уже не контролируете своё тело, ваше сознание туманно, но все еще существует. Отказ всех отделов мозга, отвечающих за сенсорное восприятие, и вот вы замкнуты в мучительной тьме и тишине, но сознание все еще существует. Отказ большинства мыслительных центров, вы овощ не только внешне, но и умственно, но, пусть и на малую толику, сознание все еще существует...

          Т.е. да, возможно, наше сознание будет существовать субъективно вечно, вопрос лишь в том, к какому итоговому состоянию этого самого сознания на бесконечности вероятностно сходятся варианты вселенных, где оно продолжает существовать. И не является ли это состояние невообразимо худшей альтернативой тому, чтобы перестать существовать. Т.е. есть все основания предполагать, что такое бессмертие окажется сильно хуже смертности, и что самое печальное, крайне вероятно, что теория таки верна, и что эта мучительная вечность объективно неизбежна.


          1. Bronx
            23.08.2021 08:39

            Одна из причин всячески способствовать продвижению биомедицины и трансгуманизма — надежда, что если теория права, и ты действительно застрянешь в таком овощном состоянии, то в твоей версии мира быстрее появятся технологии, способные вытащить тебяиз этого персонального ада (а лучше если не дадут попасть в него).


  1. kunix
    22.08.2021 16:58
    +3

    Вот эта идея с введением наблюдателя с сознанием - она плохо пахнет.
    Чем плох "наблюдатель" в виде примитивной квантовой системы?
    Зачем нам полноценный человек с мозгом?
    Неужели без наблюдателя физические законы работают иначе?

    Эти идеи берут на вооружение всякие околонаучные фрики и на выходе получается всякий "квантовый мистицизм" или даже "квантовый солипсизм", который пахнет уж совсем плохо.


    1. mayorovp
      22.08.2021 17:49
      +2

      Вселенной наблюдатель и правда не нужен, а вот нам такой "наблюдатель" нужен чтобы объяснить то что мы видим.


      1. kunix
        22.08.2021 18:00

        Я не понимаю, зачем он нужен.
        Это просто удобная метафора?
        Или реально нужен наблюдатель с разумом?

        Вообще почитал тут вики и немного охренел
        https://ru.wikipedia.org/wiki/%D0%9A%D0%B2%D0%B0%D0%BD%D1%82%D0%BE%D0%B2%D1%8B%D0%B9_%D0%BC%D0%B8%D1%81%D1%82%D0%B8%D1%86%D0%B8%D0%B7%D0%BC
        Так, Антон Цайлингер с коллегами в своей статье «Экспериментальная проверка нелокального реализма», опубликованной в журнале Nature в 2007 году, писали следующее:

        «Основная масса современных ученых являются твердыми сторонниками концепции „реализма“. Сторонники этой точки зрения считают, что наблюдаемая („внешняя“) реальность существует независимо от факта наблюдения. Однако, квантовая физика камня на камне не оставила от некоторых из наших основополагающих представлений. Теорема Белла гласит, что неверна любая теория, которая основывается одновременно на реализме и локальности. Эксперименты с квантово-сцепленными парами частиц многократно подтвердили это. Таким образом, фундаментальная концепция реализма может быть сохранена только введением нелокального дальнодействия. В настоящей работе мы показали теоретически и подтвердили результатами экспериментов, что внушительное множество таких теорий нелокального реализма, а они кажутся вполне приемлемыми, не соответствует, однако, квантовым корреляциям, наблюдаемым в эксперименте. Мы регистрировали такие корреляции между двумя квантово сцепленными фотонами, которые ранее ещё никем не были проверены. Мы показали, что такие корреляции нарушают неравенства Леггетта, предложенные им для нелокальных реалистических теорий. Наши результаты дают основания предположить, что отказ от концепции локальности может быть недостаточен для соответствия результатам квантовых экспериментов и что определённые интуитивно-принимаемые свойства реализма должны также быть отвергнуты.»[17]

        Ричард Конн Хенри (Richard Conn Henry) и Стефен Р. Палмквист (Stephen R. Palmquist), в своем комментарии к упомянутой работе, напечатанном в журнале неоднозначной репутации Journal of Scientific Exploration[en], отмечают следующее: «Теперь мы начинаем понимать, что квантовая механика может исключить любую возможность существования реальности, независимой от сознания. Она уже запрещает любую реальность, похожую на наши обычные представления о таковой. Алэн Аспэ (Alain Aspect) отмечает: „Это подразумевает, что нужно отказаться от любого такого реализма, который мог бы мне понравиться“. … И если разум не является производным от материи, а скорее является создателем иллюзии материальной реальности (что, на самом деле, в противоположность [воззрениям] материалистов, является правдой, известной с открытия квантовой механики в 1925 году), тогда теистический взгляд на наше существование становится единственной рациональной альтернативой солипсизму»[18][значимость факта?].


        1. mayorovp
          22.08.2021 18:15

          Зависит от рассматриваемого вопроса. К примеру, если мы пытаемся ответить на вопрос "почему наблюдатель A видит X" — то способность наблюдателя видеть в вопросе постулирована.


          1. kunix
            22.08.2021 18:17

            Это не ответ на вопрос.

            "Видеть" может даже система из одного атома.

            Прилетел фотон - повысился энергетический уровень.

            PS. Кто блин в карму сразу минусует? А поговорить? :)


            1. mayorovp
              22.08.2021 18:20
              -2

              Когда вы говорите, что атом "видит" — вы тем самым в своих рассуждениях одушевляете атом, делая его "разумным" наблюдателем.


              1. kunix
                22.08.2021 18:23

                Я никого не одушевляю.
                "Видеть" или "провести измерение" - значит провзаимодействовать с измеряемой системой и в результате этого изменить свое состояние.

                Зачем тут сознание?
                Введение сознания в квантовой механике - это очень серьезная заявочка с далеко идущими последствиями. Достаточно почитать цитаты из википедии выше, чтобы увидеть, куда оно все идет.


                1. mayorovp
                  22.08.2021 18:27
                  -1

                  С точки зрения русского (и не только) языка — одушевляете.


                  1. kunix
                    22.08.2021 18:31
                    +1

                    Я-то думал, у нас разговор о квантовой механике, а не о русском языке.

                    Измерительный прибор - это физическая система, которая изменяет свое состояние в результате измерения.

                    Ей не нужно сознание, чтобы провести измерение.

                    Сознание может увидеть результат измерения уже потом, через десятки секунд. А может и не увидеть, если его рядом нет.

                    Итак, повторюсь еще раз, зачем в квантовой механике сознание?


                    1. mayorovp
                      22.08.2021 18:36

                      У нас разговор о конкретном посте на хабре, который написан русским языком. И в этом тексте вы где-то вычитали про сознание, вот я и объясняю откуда оно там взялось.


                      А квантовой механике сознание не нужно.


                      1. kunix
                        22.08.2021 18:43

                        А квантовой механике сознание не нужно.

                        Внимательно прочитайте цитаты Антона Цайлингера и двух философоф выше. Эти ребята тут типа квантовую революцию устраивают. Веру поверяют наукой, ага.

                        У нас разговор о конкретном посте на хабре, который написан русским языком. И в этом тексте вы где-то вычитали про сознание, вот я и объясняю откуда оно там взялось.

                        Ну спасибо :)
                        Я прочитал не только сам пост, но и много чего по ссылкам.


                      1. DmitriyN
                        22.08.2021 18:50

                        А вы сравните, что говорит первый, и что - вторые. Отсутствие локального реализма никак не влечет за собой необходимость сознания-наблюдателя, "квантового разума" и прочих слабоопределимых философских концепций.

                        Кроме того, в научном дискурсе понятие "реализм" имеет вполне определенный четкий смысл.


                      1. kunix
                        22.08.2021 18:54

                        Ну первый нормальный ученый, хотя есть у него одна подозрительная фразочка, которую можно по-разному интерпретировать:

                        Сторонники этой точки зрения считают, что наблюдаемая („внешняя“) реальность существует независимо от факта наблюдения.

                        Чем отличается реализм в квантмехе от метафизического реализма, я примерно понимаю.

                        А вот два философа это просто фрики.
                        И таких фриков нынче много.
                        Поэтому я и возбудился из-за введения наблюдателя, обладающего сознанием.


                      1. DmitriyN
                        22.08.2021 19:07
                        +1

                        Проблема в том, что мы пытаемся использовать язык, который крайне плохо предназначен для описания того, что мы пытаемся описать. Под существованием наблюдаемой реальности стоит понимать существование определенных значений наблюдаемых, а под фактом наблюдения - некий сложный процесс декогеренции.

                        Собственно, MWI и хороша тем, что не требует ввода никаких лишних сущностей. Скажем, классический копенгаген утверждает, что есть какой-то акт наблюдения, не уточняя, что это.

                        Что же касается разума, теизма и прочего - это имхо просто попытки натянуть сову на глобус - нашу обезьянью интуицию о том, как должен работать мир на то, как он реально работает.


                      1. kunix
                        22.08.2021 21:41

                        Проблема в том, что мы пытаемся использовать язык...

                        Именно. Мы пытаемся описать словами решения уравнения Шредингера, которое несоизмеримо сложнее любых человеческих слов. И это при том, что уравнение Шредингера еще не описывает квантовый мир in its full complexity.

                        Что же касается разума, теизма и прочего - это имхо просто попытки натянуть сову на глобус - нашу обезьянью интуицию о том, как должен работать мир на то, как он реально работает.

                        Про обезъянью интуицию - как с языка сняли, тоже использую подобную "терминологию" :)

                        Тут еще есть такой феномен, что содержимое черепной коробки некоторых лысых обезъян работает по принципу I want to believe. Ищутся любые зацепки, это все компилируется в чудаковатую картину мира и вываливается на ни в чем не повинных пользователей интернета. А некоторые и статьи в журналы даже пишут.


                      1. mayorovp
                        22.08.2021 19:06
                        +1

                        Так ведь ссылки на Антона Цайлингера и квантовый мистицизм тут только вы приводили...


                    1. mayorovp
                      23.08.2021 23:21

                      В момент измерения (который совершенно не момент) детектор запутался с частицей. После этого момента мы можем сказать о начале разделения миров. При этом ни детектору, ни законам физики разделение миров не требуется, в отличии от копенгагенской интерпретации.


                      1. mayorovp
                        24.08.2021 00:19

                        детектор скажет о разделении миров уже находясь в какой-то одной из ветвей(иначе мы бы намеряли все возможные исходы со 100% вероятностью, чего не происходит)

                        Но, с точки зрения ММИ, детектор действительно "намеряет" все возможные исходы, оказываясь в результате измерения в смешанном состоянии.


                        Детектор не находится только в одной ветви, он находится в каждой из ветвей.


                      1. mayorovp
                        24.08.2021 02:37
                        +1

                        Если он действительно так делал, то мы могли бы это наблюдать.

                        Неа, не можем мы это наблюдать. Потому что мы тоже в смешаном состоянии, и каждая половинка нас видит только половину детектора.


                      1. mayorovp
                        24.08.2021 12:21

                        то он мог бы это зафиксировать физически(тоесть он мог бы зафиксировать нахождение в суперпозиции)

                        Но он не находится в суперпозиции, он находится в смешаном состоянии.


                        и мы бы смогли это пронаблюдать просто прочитав показания детектора. Этого очевидно не происходит

                        Не очевидно.


                      1. mayorovp
                        24.08.2021 13:34
                        +1

                        Обоих вариантов быть не может, потому что измерение УЖЕ произошло, вы сами это сказали.

                        Не вижу связи.


                        Хотя зачем в данном конкретном случае использовать матрицы плотности я не очень понимаю.

                        Причём тут матрицы плотности?


                        Ну, если вам это не очедно, то вы, вероятно, можете продемонстрировать показания детектора находящегося в суперпозиции\смешаном состоянии?

                        Что вы понимаете под продемонстрировать? Будучи запутанными с детектором, мы можем "увидеть" только одну из веток — ту, в которой находимся.


                      1. mayorovp
                        24.08.2021 15:07
                        +1

                        Так почему допущение-то, если так по формулам получается?


                      1. red75prim
                        30.08.2021 20:57
                        +1

                        По формулам получается (при некоторых предположениях о волновой функции прибор-наблюдатель), что система частица-измерительный прибор-наблюдатель декогерирует - распадается на несколько частей, которые не могут взаимодействовать.

                        Копенгагенская интерпретация дополнительно предполагает, что все части результирующего квантового состояния, кроме одной, пропадают. Неизвестно когда именно пропадают, неизвестно из-за чего пропадают.

                        В ММИ неизвестно другое - почему наблюдатель обнаруживает себя в некотором состоянии с вероятностью пропорциональной квадрату его амплитуды (правило Борна).

                        Так что хоть формулы одинаковы, но эти интерпретации вызывают разные вопросы.


                      1. 0serg
                        31.08.2021 18:40

                        В ММИ неизвестно другое — почему наблюдатель обнаруживает себя в некотором состоянии с вероятностью пропорциональной квадрату его амплитуды (правило Борна).

                        Там как раз все прекрасно известно. Декогеренция приводит к тому что изначально запутанное состояние превращается в случайную смесь не-запутанных состояний, причем количество разных состояний в смеси определяется правилом Борна. К примеру в известных экспериментах с двумя щелями если фотоны проходящие через две щели остаются запутанными то возникает интерференционная картина (каждый фотон одновременно «проходит через обе щели»), а вот если добавить условия обеспечивающие декогеренцию фотонов до того как они дойдут до экрана то интерференционная картина исчезает — поток фотонов превратился в случайную смесь из фотонов проходящую через левую щель и фотонов проходящих через правую щель. В трактовке ММИ наблюдатель является частью этой случайной смеси. Предположим к примеру что в нашем эксперименте щели не равные и 25% фотонов проходит через левую щель а 75% через правую. Запустим одиночный фотон. Исходя из формул квантовой механики после декогеренции мы с вероятностью 25% обнаружим фотон прошедшим через левую щель, с вероятностью 75% — через правую. С точки зрения ММИ в момент декогеренции 25% наблюдателей оказались в мире где фотон прошел через левую щель а 75% — в мире где фотон прошел через правую и это описывается ровно теми же формулами которые традиционно интерпретируется как то что 25% фотонов прошли через левую щель и 75% через правую.


                      1. red75prim
                        31.08.2021 19:50

                        Что значит 25% наблюдателей, если их всего два (один видит фотон слева, другой - справа)?

                        Имеется в виду many minds интерпретация (в которой в каждом теле находится бесконечное число наблюдателей, часть из которых попадает в один бранч, а часть в другой)? Как-то слишком эзотерично. Никак не объясняется что такое "наблюдатель", их бесконечное количество и почему они ведут себя именно так.


                      1. 0serg
                        01.09.2021 13:26

                        Наблюдатель — это физический объект описываемый теми же самыми уравнениями что и фотоны. Как фотон может находиться в состоянии «25% что это фотон прошел через левую щель, 75% что через правую» так и наблюдатель может находиться в состоянии «25% что он видит что фотон прошел через левую щель, 75% что через правую». Там одно и то же уравнение просто примененное для описание состояния разных систем. ММИ собственно в известном смысле постулирует что ничего кроме этого смешанного статистического состояния собственно и нет. Для фотонов мы его интерпретируем как то что 25% фотонов проходит через левую щель, 75% через правую. А для наблюдателя естественной трактовкой того же самого результата будет именно считать что 25% наблюдателей видит как фотон проходит через левую щель, 75% через правую.


                      1. red75prim
                        01.09.2021 14:24

                        Я так и не понял, что такое 25% от двух вариантов наблюдателя. Но если вы понимаете, возможно стоит опубликоваться в каком-нибудь физическом журнале. Так как до сих пор нет общепринятого вывода правила Борна в рамках ММИ. См., например, https://www.math.ru.nl/~landsman/Born.pdf


                      1. 0serg
                        01.09.2021 20:52

                        По Вашей ссылке написано что хорошего «вывода» правила Борна нет ни в одной интерпретации (оно обычно просто постулируется). Там же написано что ряд теорий включая ММИ утверждают что им удалось это сделать, даже ссылка дается на S. Saunders: Derivation of the Born rule from operational assumptions. Просто там нет полной ясности действительно ли это независимый вывод правила Борна или же эти рассуждения на самом деле скрыто опираются внутри себя на все тот же постулат просто переформулированный в другом виде.

                        Что до 25% наблюдателя то это вообще говоря просто формула. Собственно теория на этом заканчивается, дальше начинается ее интерпретация. Мы знаем что такая же формула применительно к фотонам естественным образом интерпретируется как то что 25% фотонов в одном состоянии, 75% в другом. ММИ говорит что мы можем рассматривать этот результат как если бы существовало бесконечно много миров и в 25% из них наблюдатель видел один результат а в 75% — другой. Оно не говорит что это действительно так, просто констатирует что в этом случае формула имеет естественную интерпретацию.


                      1. red75prim
                        31.08.2021 22:48
                        +1

                        "что каждый математический объект имеет некоторое отражение в "реальности" - самом по себе, вообще говоря, очень сильное предположение.

                        А это не "каждый математический объект", это объект теории, которая пока не опровергнута.

                        Вероятностная функция, которая не имеет физического отражения и никуда реально не коллапсирует, как думают многие.

                        Но если измерим состояние частицы после измерения, то всё будет выглядеть так как будто она сколлапсировала (или декогерировала и мы видим только одну часть). Или квантовый эффект Зенона - мера нашего незнания заставляет ядро не распадаться?

                        Так что могу вернуть вышесказанную фразу: "то, что мера нашего незнания - это фундаментальная физическая характеристика, вообще говоря, очень сильное предположение".


                      1. red75prim
                        31.08.2021 23:15

                        Когда вы сделали измерение ваше неопределенность уменьшилась до 0

                        Это в простейшем случае. Если измерять состояние одной частицы из запутанной пары, то неопределенность не уменьшится до нуля пока не придёт сообщение о результате измерения состояния второй частицы. Так что? Включать в фундаментальную теорию все способы обмена информацией об измерениях?


                      1. DrSmile
                        24.08.2021 02:08
                        +3

                        Эти два высказывания логически эквивалентны. Так если они эквивалентны в чем приимущество ММИ-то?
                        В том, что в копенгагене коллапс объективный, а в ММИ — субъективный. Соответственно, нету никаких проблем с неопределенностью времени и условий коллапса, с его сверхсветовым распространением. Я, вообще, не понимаю, как копенгагенскую интерпретацию можно воспринимать всерьез: она даже не является физической теорией, ибо условия наступления и протекания коллапса, в принципе, не определены. Между тем, то, что эволюция по Шредингеру приводит к возникновению параллельных миров, показывается элементарно. Можно спорить по поводу разных тонкостей, но вот само существование миров прямо следует из уравнений. Так что разница между ММИ и копенгагеном — это разница между строгой физической теорией, поддерживаемой бритвой Оккама, и полуфилософским набором утверждений, придуманных на заре развития квантовой механики чтобы хоть как-то интерпретировать результаты.


                      1. DrSmile
                        24.08.2021 13:13
                        +1

                        У вас, по большей части, возражения философские, а не по существу. Но, так и быть, поясню свою мысль.

                        Коллапс в копенгагене — это не коллапс некого реально существующего объекта
                        Коллапс в копенгагене — это реальный физический процесс в рамках соответствующей теории. «Реальными» в данном случае я называю математические объекты теории, безотносительно их реальности в настоящем мире. То, что даже пропоненты копенгагенской интерпретации видят ее проблемы и считают ее плохо соответствующей реальности, скорее подтверждает мою мысль.
                        Она приводит нек возникновению параллельных миров, а к возникновению некоторых векторов, которые между собой не взаимодействуют
                        Эти векторы называются «мирами» просто по определению. С точки зрения теории они реально существуют (а что там в нашей реальности — никто не знает и знать не может). А подтверждать, согласно бритве Оккама, надо не присутствие параллельных миров, а их отсутствие, ибо они прямо следуют из уже подтвержденных уравнений.
                        Это всего лишь математический формализм, которому вы с потолка пытетесь придать рельное существование.
                        Математический формализм — это все, что, в принципе, может иметь смысл. Если наша вселенная является сколь-нибудь логичной, то это значит, что некоторый математический формализм реально существует. В физике основной критерий реальности теории — соответствие эксперименту. У уравнения Шредингера (на самом деле, у функциональных интегралов КТП) с экспериментальными подтверждениями проблем нет, поэтому все, что оно описывает, считается (в некотором смысле) реальным по определению.
                        Если уж гоорить о бритве Оккама, то самым правильным будет не выдумывать вообще никакую интерпретацию
                        Да, примерно так и получается ММИ.
                        которая постулирует физическое существование параллельных миров, которые, внимание, нельзя принципиально пронаблюдать
                        Еще раз, ММИ никаких параллельных миров не постулирует. Она постулирует уравнение Шредингера. Возникновение миров — это уже следствие из уравнений, с которым она не пытается бороться, в отличие от копенгагена, который как раз ради этого вводит разные нефизические постулаты.
                        Волновая функция н физична, как вы тут пытались представить — это просто инструмент расчетов.
                        Вся физика на 100% — это инструмент расчетов. Нету никаких данных свыше «истинно реальных» теорий, одни инструменты. Самые успешные из этих инструментов считаются реальными просто по определению, для удобства.


                    1. DmitriyN
                      24.08.2021 02:01
                      +2

                      Я, честно говоря, не уверен, что именно утверждается в эталонных ММИ и копенгагене из палаты Мер и весов, но я и те мои коллеги, которые вообще задумывались об этом, считают следующее:

                      1. Есть интерпретации, авторы которых занимаются борьбой с квантовостью путем введения диссипативных факторов в квантовую динамику (OrchOR, GRW и иже с ними). Это, скорее не интерпретации, а попытки модификации теории с целью сделать ее более классической.

                      2. Интерпретации, постулирующие классическую природу "наблюдения". Это классический копенгаген и, например, consistent histories (последняя - это "непрерывная" версия КИ). Насколько я понимаю, они постулируют некую "фиктивность" стейт-вектора, принимая его за вычислительный инструмент (я не совсем понимаю, что это значит - об этом ниже) и вводят дополнительную процедуру измерения, выполняющую роль барьера между вычислительным квантовым описанием и классическим наблюдением (в CH нет конкретной процедуры измерения, но там есть необходимость ввести континуум проекторов, играющих ту же роль). Где именно происходит измерение или как именно выбирается эта система проекторов заметается под ковер так называемой measurement problem. Я не уверен, но мне кажется, это не совсем та картина, которую вы описываете.

                      3. ММИ, которая не постулирует ничего, кроме стандартного формализма квантовой механики. Также ММИ предполагает, что стейт-вектор имеет некоторую объективную реальность (опять же, что бы это ни значило) и что вся вселенная описывается большим компаунд-состоянием (или, хотя бы матрицей плотности). Название "многомировая" и следующие из него картинки с разделяющимися котами - это просто плохой нейминг. В большинстве случаев, как только информация о состоянии какой-либо микроскопической квантовой системы достигла макроскопических масштабов, запутанность начинает бесконтрольно распространяться и интерференция состояний становится невозможной. Это то, что обычно называют "расщеплением миров", хотя мне эта терминология, как я уже сказал, не очень нравится.

                        По поводу того, что имеет право называться "имеющим объективную реальность". Честно говоря, я не уверен, что у кого-то есть ответ на этот вопрос. На мой взгляд, объективную реальность может иметь либо некая конечная теория, которая описывает все, которая не факт, что существует, либо то, что в данный момент вам гносеологически удобно. Вот мне удобно считать стейт-вектор "объективно реальным". Баллистику, рассчитывающему космическую миссию вообще удобно, например, считать объективно реальной нерелятивистскую механику, которая точно не является корректным описанием вселенной и никому от этого не плохо.

                        Повторюсь, ММИ не постулирует никакое существование множественных миров и процесса расщепления. Она постулирует только постоянное существование глобального квантового состояния. Множественные миры - это просто некая аналогия для кусков стейт-вектора, потерявших возможность интерферировать друг с другом из-за декогеренции. Безусловно, декогеренция не решает, например, проблемы с правилом Борна, хотя стоит отметить, что некие подвижки в эту сторону есть, напр. https://arxiv.org/pdf/1405.7907v3.pdf. Но другие интерпретации (кроме имхо шизофренических интерпретаций объективного коллапса) ее тоже не решают.

                      Так как происходит переход от такого вот математического формализма с лучем в гильбертовом пространстве к постулированию физического существования множественных миров? Математичесий формализм != физическая реальность и существование векторов-состояний никак не говорит о том,
                      что состояния, которые они описывают реально существуют.

                      Было бы очень неплохо, если бы кто-нибдь уточнил, что значит "реально существуют" и чем это отичается от математического формализма. Боюсь на этот вопрос можно будет ответить только если будет построена окончательная теория и будет железобетонное доказательство того, что она действительно окончательная (сомнительный вариант развития событий).

                      Никакого перехода от квантовости к классике или обратно нет - это просто следствие неправильного понимания копенгагенской интерпретации.

                      Очень странное утверждение. Теорема Эренфеста, эйконал и т.п. в одну сторону, деформационное квантование - в другую.

                      Это просто констатация того, что мы, проводя измерение фиксируем некое состояние

                      Это просто следствие существования pointer states. Именно что в копенгагенской интерпретации это беспричинная констатация факта.

                      Правильно ли я понимаю, что в вашей интерпретации ММИ никаких параллельных "миров" физически не существует? Если так то в чем принципиальное отличие вашей интерпретации ММИ и копенгагена?

                      Неинтерферирующие сектора глобального стейт-вектора существуют (ну или не глобального, а стейт-вектора достаточно крупномасштабной системы). Не нравится название "мир" - можете называть как-нибудь по-другому (кстати, как я сказал, мне тоже не нравится).

                      Зачем под диагонализацией матрицы понимать процесс расщипления, если его нельзя продемонствировать экспериментально?

                      Ну что значит нельзя. Я вот могу взять систему из трех-четырех кубитов и экспериментально продемонстрировать процесс расщепления их состояния на пару неинтерферирующих. Или вам прям над всей вселенной хочется эксперимент поставить? К сожалению мы так не умеем.

                      Скажите - вы согласны, что физика, как наука должна подтверждать свои предсказания экспериментом или это по вашему не обязательно?

                      Нет, не согласен. Видите ли, теории, возникшие раньше обладают неким несправедливым преимуществом. Когда люди говорят о верификации новой теории, они подразумевают не только верификацию корректности относительно массива имеющихся данных, а еще и о неких новых предсказаниях, которые не должны описываться в рамках существующих теорий. Проблема в том, что если бы история шла по-другому и более поздняя в рамках нашей истории теория (2) появилась раньше более ранней (1), то пропоненты фальсифицируемости говорили, что теория (1) - фигня, потому что она не верифицируема. Тем не менее, она вполне верифицируема, так как описывает пачку наблюдаемых данных.

                      Новая теория так же может включать в себя ненаблюдаемые вещи, которые принципиально наблюдать нельзя. В этом нет абсолютно ничего плохого - вас же не смущает, например, калибровочная инвариантность в классической электродинамике? В КТП вон вообще есть куча калибровочных инвариантностей и (о ужас) ренормализация.

                      ИМХО основной профит от теорий - это необычный взгляд на описание мира. Безусловно при этом требование непротиворечивости никуда не девается.

                      Представим, что у нас детектор, детектор производит измерение. До измерения декогенерации не произошло и детектор и частица в одном "мире", после она уже произошла и у нас появилось два детектора и две частицы. Возникает вопрос: а что было собственно в момент измерения? Что намерял детектор? Вообще все состояния сразу со 100% вероятностью и они уже после этого разъехались? Но почему мы этого не наблюдаем? Или детектор мы рассматриваем как девайс в какой-то одной уже отдельной ветви и все вычисления делаем для него?

                      Ну, на всякий случай повторюсь. Это не два детектора и две частицы. Это два сектора состояния, которые друг с другом уже не могут взаимодействовать.Что было в момент измерения можно до какого-то предела в простых системах посчитать относительно точно интегрируя master equation.


    1. TimeCoder
      29.08.2021 22:02
      -2

      Измерительная система регистрирует событие, наблюдатель (человек) после некоторой задержки смотрит показания. Наблюдение невозможно без наблюдателя (внезапно), и здесь в принципе нет, и не может быть способа проверки состояния прибора без наблюдения сознанием. Да, не стоит всерьез верить в то, что табуретки пляшут, пока на них никто не смотрит (хотя опровергнуть это невозможно). Но сознание есть, это факт, который странно игнорировать. При этом объяснить его природу пока никто не смог. Наблюдение и измерение - понятия весьма близкие, и не понимая природу сознания, провести между ними четкую черту пока нельзя. Если в квантовом мире состояние системы меняется при измерении, но результат измерения невозможно узнать без его наблюдения сознанием, а природа сознания не понятна, и существуют попытки объяснить работу сознания опять же квантовыми эффектами - убрать из этой схемы сознание наблюдателя - значит потерять объективность (парадоксально, да).


      1. kunix
        29.08.2021 22:30
        +1

        Это все звучит довольно бредово.

        Тут отписывались нормальные спецы по моим вопросам.

        Можно почитать и проникнуться. Никакой эзотерики там таки нет, как и ожидалось.


        1. TimeCoder
          30.08.2021 00:17
          -2

          Где в моих словах вы увидели эзотерику? Для вас всё, что связано с сознанием - эзотерично? Идея вынести его за скобки, и рассматривать мир так, словно сознания не существует - не оригинальна, и давно требует уточнения. Вам есть что сказать по делу, особенно про природу сознания (убрать которое из вопроса вы так хотите)? Или только «бред» и минусы ставить?


          1. kunix
            30.08.2021 07:23
            +1

            Я увидел некоторые мысли, из которых потом вырастает квантовый солипсизм и вся подобная эзотерика.
            Законы природы работают одинаково до моего появления на свет, когда я сплю, когда я пьян/накурился, и когда я сдохну тоже будет работать все так же.
            Меня можно заменить на очень сложную нейросеть, и тоже ничего не изменится.

            В целом, что сознание - это продукт действия материи, которая в черепной коробке. Почему эта материя в принципе должна что-то там требовать от физики?

            Минус вам поставил кто-то другой.
            1) Я сам минусы никогда не ставлю.
            2) А теперь еще и не могу ставить, спасибо молчаливым минусаторам за отрицательную кармочку.


            1. TimeCoder
              30.08.2021 09:41
              -2

              Строго говоря, абсолютно (даже примерно) не понимая, что такое сознание, нельзя говорить о том, влияет ли оно на физику. Возможно, именно сознание вызывает коллапс волной функции. Но я понял, ок, вас не переубедить.

              p.s. Добавил кармы.


              1. kunix
                30.08.2021 09:45
                +1

                За карму спасибо.

                Но блин, вы же понимаете, на какую скользкую дорожку встаете?
                Еще 100 лет назад никто не знал о волновых функциях.
                А законы физики уже работали вовсю.


                Это метафизический реализм, на этом держится вся фундаментальная наука.
                Нужны очень серьезные основания, чтобы от него отказаться.

                Если у вас есть ссылки на хорошие рецензируемые журналы, где вот прямо серьезно рассматривается вопрос сознания в квантовой физике, я с удовольствием почитаю. Но я, как человек не далекий от физки, очень сомневаюсь. Это противоречит моим убеждениям покруче летающих зеленых человечков.


              1. mayorovp
                30.08.2021 11:36
                +2

                Возможно, именно сознание вызывает коллапс волной функции.

                С точки зрения ММИ — коллапса волновой функции не существует.


                1. mayorovp
                  23.08.2021 12:02

                  Она не эквивалентна для случая бесконечного числа состояний. Просто потому что вообразить волновую функцию в гильбертовом пространстве мы можем, а бесконечных матриц не существует.


                1. DmitriyN
                  23.08.2021 16:30
                  +2

                  Знаете, мне кажется мы говорим примерно об одном и том же, за исключением того, что вы держите в голове какую-то конкретную квантовую теорию. Но квантовая механика - это не какая-то конкретная теория, а шаблон, ровно как и классическая - не теория описывающая движение определенных шариков, а просто подход к описанию динамических систем.

                  Поэтому ваше замечание про то, что переход от, скажем, теории эпициклов к теории Ньютона это то же самое, что переход от CM к QM выглядит несколько странным. QM - это просто способ построения динамических теорий с расслабленным реализмом (читай, некоторые параметры системы могут быть не определены). Безусловно, КМ может быть не окончательным стилем описания, но в любом случае, классическим он уже точно не станет.

                  Ваше утверждение о том, что КМ - чисто статистическая теория в корне не верно. Так было когда люди не научились управлять когерентными системами. Сейчас же можно тупо сделать относительно крупномасштабную квантовую систему и убедиться, что предсказания, которые мы получаем действительно выглядят так, как будто квантовое состояние объективно существует. Вы можете провести систему по сложной контролируемой траектории (в промежутоных точках которой правило Борна даст вам нетривиальное статистическое распределение) и убедиться, что она попала ожидаемое состояние. Вы можете возразить, что это все равно статистическое предсказание, но тогда возникает вопрос - когда именно оно становится статистическим. В классическом копенгагенском подходе ответ на этот вопрос заключается в том, что мы не знаем. Есть некая магическая процедура наблюдения, приводящая к коллапсу.

                  В ММИ, которая имхо и является теорией стиля "заткнись и считай", есть аналогичная процедура. Называется она "постселекция" - это когда нам численно неудобно отслеживать все состояние целиком и мы просто искусственно отбрасываем куски состояния. То есть это просто расчетный инструмент, а не некий объективно существующий процесс.

                  Ваш последний вопрос говорит о том, что, скорее всего, вы не очень хорошо знаете КМ. Когда сейчас говорят про "волновую функцию" никто обычно не имеет ввиду именно функцию координат. Обычно говорят просто про некий элемент (точнее луч) в гильбертовом пространстве. И волновая функция а-ля Шредингер, и матричная механика, и пространства Фока являются частным случаем этого. В матричной механике слагаемые такие же, как и везде - суммы тензорных произведений состояний подсистем.

                  PS. Теория декогеренции вполне описывает процесс расщепления, если под расщеплением понимать диагонализацию редуцированной матрицы плотности какой-то подсистемы.


            1. 0serg
              23.08.2021 09:46
              +2

              Вы описываете то что называется «детерминистической квантовой механикой основанной на скрытых параметрах». Т.е. есть некие скрытые от нас параметры, они как-то эволюционируют, а мы можем только статистически наблюдать отдельные моменты этой эволюции. Так вот эта гипотеза доказанно неверна. Ее перепроверяли множество раз самыми разными способами, но увы, все они доказали то что волновая функция «реальна» в том смысле что она точно не является лишь способом статистического описания некоего скрытого состояния (скажем положения частицы).


        1. DmitriyN
          22.08.2021 20:31
          +3

          Давайте сразу отделим мух от котлет. В каком-то смысле, на уровне махания руками, ММИ две (ну, точнее два уровня ее понимания). Первый уровень, предложенный Эвереттом, просто утверждал, что нет никакого деления мира на классический и квантовый, как в копенгагенской интерпретации, а динамика всегда квантовая, просто после того, как наблюдатель (тоже квантовый), измерил объект, у нас теперь есть сумма двух состояний системы наблюдатель+объект, в каждом из которых наблюдатель "видит" объект в определенном состоянии. Реальное состояние системы - это все еще сумма, но любая динамика наблюдателя уже будет такой, как если бы состояние системы было определенным. Это просто тупо следствие кинематики. Кто-то из авторов этого утверждения (не уверен, что сам Эверетт), решил назвать компоненты этой суммы "мирами" и завертелось. Собственно, на этом уровне понимания все популяризаторские материалы и остаются, несколько его упрощая и рассказывая про распад вселенной на куски.

          Кроме нашей системы, (наблюдатель + наблюдаемый объект), во вселенной определенно есть что-то еще, но пока это что-то еще не провзаимодействовало с нашей системой, оно, соответственно, на сумму состояний не распалось. Поэтому неправильно говорить, что любой исход делит именно всю вселенную на две.

          Второй уровень понимания - это современная теория декогеренции, которая описывает, как именно такой процесс происходит. Например, за счет чего в больших системах состояния стремятся к сумме таких, что они начинают выглядеть как классические. Это довольно сложная теория и там еще куча открытых технических вопросов, тем не менее, она вполне объясняет, как именно системы начинают выглядеть классически из-за взаимодействия с "термостатом" (большим числом степеней свободы). При этом, предсказываемая динамика именно такая - был объект в суперпозиции + наблюдатель, получилась суперпозиция из (наблюдатель + объект) в различных состояниях. При этом, совершенно не важно, что такое наблюдатель - это может быть, например, кусок коаксиального кабеля в эксперименте со сверхпроводящим квантовых компьютером. Потом с этим кабелем повзаимодействует какой-нибудь усилитель, после чего они втроем будут образовывать сумму, потом экспериментатор и так далее.

          А работа квантовых компьютеров здесь при том, что если рассматривать ММИ так, как ее преподносит большинство популярных материалов, то она либо превращается обратно в копенгагенскую с ее мистическим процессом наблюдения, либо ломает КМ, делая работу квантовых компьютеров и вообще крупномасштабную квантовую когерентность невозможной.

          PS. Конечно, теория декогеренции была еще тогда, когда Эверетт придумал ММИ (собственно она его на это и сподвигла, насколько я понимаю), но уровень понимания сейчас сильно другой.

          Тем не менее если вы пропонент ММИ, то хотелось бы получить какое-то объяснение в рамках ММИ. Это совершенно валидный вопрос.

          Это как просить объяснить движения фондового рынка с помощью гидродинамики. Еще раз, эта проблема вообще никак с ММИ не связана.


  1. anonymous
    00.00.0000 00:00


    1. kunix
      22.08.2021 17:46

      Насколько я понимаю, ММИ заменяет случайные события случайным выбором вселенной.
      И, как было сказано в статье, ММИ не влияет на расчеты.
      Выводы из ММИ остаются теми же, как из других современных теорий.

      Мне приходит на ум утверждение креационистов - типа мир был создан 10 лет назад, причем таким образом, чтобы физические законы работали как работают, в земле были ископаемые динозавры, а у вас в голове цельные воспоминания, etc.

      Я почитал ВИКИ и таки нашел некий эксперимент, который позволит проверить ММИ.
      https://en.wikipedia.org/wiki/Many-worlds_interpretation
      Testability

      In 1985, David Deutsch proposed a variant of the Wigner's friend thought experiment as a test of many-worlds versus the Copenhagen interpretation.[32] It consists of an experimenter (Wigner's friend) making a measurement on a quantum system in an isolated laboratory, and another experimenter (Wigner) who would make a measurement on the first one. According to the many-worlds theory, the first experimenter would end up in a macroscopic superposition of seeing one result of the measurement in one branch, and another result in another branch. The second experimenter could then interfere these two branches in order to test whether it is in fact in a macroscopic superposition or has collapsed into a single branch, as predicted by the Copenhagen interpretation. Since then Lockwood (1989), Vaidman and others have made similar proposals.[33] These proposals require placing macroscopic objects in a coherent superposition and interfering them, a task now beyond experimental capability.


    1. DmitriyN
      22.08.2021 19:21
      +2

      ММИ - это, фактически, "никакая" интерпретация квантовой механики - она не вводит лишних сущностей сверх обычных динамических уравнений КМ. Поэтому, по большому счету, проверять надо остальные интерпретации, для корректности ММИ достаточно просто того, что мир квантовый.

      Вообще говоря, термин Многомировая интерпретация имхо не очень удачен, потому что вызывает вот такие вот вопросы, создавая иллюзию того, что "миры" ММИ это что-то типа параллельных вселенных из фантастических произведений.

      На самом деле, даже популярное утверждение о том, что вселенная вот так просто берет и сразу разваливается на две - это ерунда, потому что информация не распространяется с произвольной скоростью. Поэтому в картинке с миром, разделяющимся на два он не делает это мгновенно, причем совершенно не обязательно со скоростью света. О чем можно судить хотя бы потому, что квантовые компьютеры худо-бедно работают.

      Что касается вопроса "почему мы не наблюдаем абсурдные вселенные". Тут КМ вообще ни при чем, парадокс больцмановского мозга от квантовости мира никак не зависит.


      1. kunix
        22.08.2021 21:48

        Ну наконец-то нормальный специалист отписался.
        А то можно потеряться тут среди мнений.

        На самом деле, даже популярное утверждение о том, что вселенная вот так просто берет и сразу разваливается на две - это ерунда, потому что информация не распространяется с произвольной скоростью.

        Тоже были такие мысли.
        Но вот разве при взаимодействии с одной из запутанных частиц, у них обоих мгновенно не коллапсирует волновая функция?
        То есть, ММИ тоже ведь должна учитывать этот эффект, и fork-ать мир мгновенно и около первой частицы и около второй? То есть сразу всюду?

        Также волновая функция одной частицы не ограничена малой областью пространства около частицы, а коллапсирует сразу всюду? Типа такое дальнодействие.

        С другой стороны - а что такое "мгновенно" при релятивизме?

        Короче, я запутался :)


        1. DrSmile
          22.08.2021 22:16
          +1

          То есть, ММИ тоже ведь должна учитывать этот эффект, и fork-ать мир мгновенно и около первой частицы и около второй? То есть сразу всюду?
          Разделение на миры в ММИ субъективно и плохо определено. Это просто слагаемые в полной волновой функции, на которые мы ее разбиваем для удобства описания. Можно представить плоскость, посередине которой есть один холм, который плавно разделяется на два. В начале холм один, в конце их два, однако определить точный момент когда 1 переходит в 2 невозможно, это происходит плавно и непрерывно.

          Еще хорошей аналогией является представлять мир в виде плоского тонкого листа, который может расщепляться на более тонкие (по настоящему, толщина еще должна быть комплексным числом). Расщепление — это плавный процесс, происходящий по мере распространения информации (запутывания) по вселенной, не превышающий скорость света. Например, в случае Шредингерова кота, вселенная сначала расщепляется в пределах изолированного ящика, и только когда экспериментатор открывает крышку, волна расщепления вырывается наружу.


          1. plus79501445397
            23.08.2021 10:00
            +2

            То есть, ММИ тоже ведь должна учитывать этот эффект, и fork-ать мир мгновенно и около первой частицы и около второй? То есть сразу всюду?
            Разделение на миры в ММИ субъективно и плохо определено.
            В свое время в ветке ответа на мой вопрос уважаемый Shkaff любезно предоставил ссылки на ряд статей по ММИ, в том числе вот на эту статью Л. Вайдмона, где говорится, что не смотря на отсутствие «дальнодействия» все же имеется некоторая нелокальность в ММИ: «Although there is no action at a distance in the MWI, it still has nonlocality. The core of the nonlocality of the MWI is entanglement which is manifested in the connection between local Everett worlds of the observers». И на примере GHZ-состояния иллюстрируется в чем это конкретно заключается.
            Имхо, если принять наличие такой нелокальности в ММИ, то остальной «механизм» создания/распространения миров в ММИ становится более определенным и многие (но не все конечно) вопросы снимаются.


            1. DmitriyN
              23.08.2021 16:49

              Это просто такая терминология. Мы можем это называть "квантовой нелокальностью", можем отсуствием "локального реализма". Кому как больше нравится. На самом деле это просто некое явление, которое ведет себя контринтуитивным образом и не имеет хорошего описания на естественном языке.

              Если понимать под локальностью именно то, что под этим понимается в ОТО, то современные квантово-полевые теории существенно локальны, просто по построению (а вот квантовая гравитация, кстати, может оказаться нелокальной именно в этом смысле).


  1. anonymous
    00.00.0000 00:00


  1. anonymous
    00.00.0000 00:00


  1. anonymous
    00.00.0000 00:00


  1. anonymous
    00.00.0000 00:00


  1. anonymous
    00.00.0000 00:00


  1. anonymous
    00.00.0000 00:00


  1. anonymous
    00.00.0000 00:00


  1. anonymous
    00.00.0000 00:00


  1. anonymous
    00.00.0000 00:00


  1. anonymous
    00.00.0000 00:00


  1. anonymous
    00.00.0000 00:00


  1. anonymous
    00.00.0000 00:00


  1. anonymous
    00.00.0000 00:00


  1. anonymous
    00.00.0000 00:00


  1. anonymous
    00.00.0000 00:00


  1. anonymous
    00.00.0000 00:00


  1. anonymous
    00.00.0000 00:00


  1. anonymous
    00.00.0000 00:00


  1. anonymous
    00.00.0000 00:00


  1. caballero
    22.08.2021 19:49
    -1

    Интересно чтобы делали все эти псевдоученые если бы не инет. Никто бы даже не слышал ни о Тегмарке ни о Вольфраме ни о прочих фриках. Не говоря уже о б заполонивших сеть эфирщиках и прочих плоскоземельщиках.

    ММИ - это не более чем очередная (после Шродингера и Эйнштейна ) попытка притащить в квантовую механику детерминизм. Впрочем сам Эверет забил на свою теорию (видимо после того как Уилер свозил его в Копенгаген и отцы-основатели КМ парню обьяснили что он просто не понимает сути КМ).

    Но физика - экспериментальная наука - за почти сто лет (через 4 года - юбилей выхода основополагающей статьи Гейзенберга) существования КМ пока нет ни одного эксперимента, не совпадающего с ее предсказаниями. В частности экспериментально подтверждено отсутствие обьективной реальности - то есть без указания наблюдателя нет вообще никакого смысла рассматривать волновую функцию а значит и эволюцию квантовой системы (в частности эксперимент с отложеным выбором того же Уиллера).

    Я уже не говорю о том что отрицая случайности ММИ не может предьявить формулу по которой можно предсказать например точный момент распада ядра радиоактивного элемента (или например момент распада свободного нейтрона).

    Можете минусовать - я привык, других аргументов у сторонников лженаучных теорий обычно не находится.


    1. 0serg
      22.08.2021 20:22
      +5

      Я не вижу в описываемой статье ни отрицания случайности, ни спора с утверждением что наблюдатель важен. Только альтернативную интерпретацию феномена коллапса волновой функции которая выглядит довольно здраво.


    1. kunix
      22.08.2021 22:20

      В частности экспериментально подтверждено отсутствие обьективной реальности - то есть без указания наблюдателя нет вообще никакого смысла рассматривать волновую функцию а значит и эволюцию квантовой системы (в частности эксперимент с отложеным выбором того же Уиллера).

      Кто у вас такой "наблюдатель"? Это разумное существо или тупо другая квантовая система?


  1. MasterOgon
    22.08.2021 20:26
    -4

    Спасибо автору за подробное изложение этих идей. Но я категорически не согласен, тысяча минусов мне в карму, с многими из них!

    1.Во первых карта никуда не упадет потому что она стоит ровно и сохраняет равновесие, либо она кривая и упадет куда ей надо.

    2. Во-вторых единственный реальный пример неопределенного дуализма , частица-волна, совсем не обязательно таковым является. Этому нет доказательств. Вихривое кольцо или солитон в вязкой среде ведут себя так же как частица волна и здесь нет никакой мистики.

    3. Нет никакого хаоса. Когда в усккорителе сталкивают частицы и открывают новые происходит такое, как если бы они разбили бутылку, дали каждому осколку название и придумывали бы свойства, за которые он отвечает - округлость, прозрачность, зеленость и тд. Это чистой воды мошенничество!

    И потом - сколько сложных концепций придумано, порой очень верных, но совершенно никакой реальной практики. И в то же время эта практика есть, но в среде, которая далека от научной, практика, которая научной средой отрицается. Дебилы!!! Если бы кто то мог применить научный метод (именно физику) к таким вещам как астрал, сновиденье, медитация, то был бы открыт грааль знаний, которые по сравнению с сегодняшним уровнем теоретиков можно было бы назвать божественным. Это я говорю как практик. Я методом тренировок и самодисциплины следуя разным древним учениям без всякой наркоты сам входил в измененные состояния сознания когда ты существуешь вне своего я осознанно. Там тебе и эффект наблюдателя, там тебе и кротовые норы, там тебе и варп, и квантовая запутанность, и безвременье, и бесконечные параллельные миры! Причем все это четко кореллируется с такими вещами как закон сохранения энергии, импульса. Больше того - эффект наблюдателя это эффект творения реальности. Все что вокруг это коллективный эффект наблюдателей. Поэтому все так незыблемо в целом и в то же время податливо в малом. А причина не в том что ее нет, а в том что она изначально необъяснима так как не имеет конца и начала. Да что тут говорить, я полетел


  1. iShrimp
    22.08.2021 20:42
    +2

    Я поддерживаю мнение, что многомировая интерпретация лучше объясняет квантовые процессы. Но я не пойму, как работает квантовая цензура и почему разные ветви событий могут идти параллельно, а могут разойтись (подвергнуться декогеренции).

    Для примера рассмотрим двухщелевой эксперимент с электроном. Вообще, электрон может полететь по любой траектории, не противоречащей законам физики. Существует бесконечное множество (континуум) вариантов развития событий. Отбросим те варианты, при которых электрон не попал в щель. Итак, электрон взаимодействует с атомами, из которых состоят края щели, и подвергается дифракции. Но почему-то ему не важно, с каким именно атомом он провзаимодействовал - все дальнейшие его пути продолжают существовать параллельно, "смешиваются" и образуют интерференционную картину. Далее, когда электрон попадает на экран, ход событий разделяется на столько вариантов, сколько атомов в экране. Так вот,

    1. почему материал, из которого изготовлен барьер со щелями, не вызывает декогеренцию и не разрушает интерференционную картину?

    2. почему материал, из которого изготовлен экран, вызывает декогеренцию? и почему мы видим на экране одну точечную вспышку, а не размазанную в форме интерференционной картины?

    Вспомним другой эксперимент, который на первый взгляд кажется логичным и понятным, - квантовый ластик. Если мы не считываем информацию о пути фотона (или считываем, но затем стираем её), то интерференция сохраняется. Если мы сохраняем эту информацию, то интерференция разрушается. Т.е., вместе с нашим фотоном Вселенная "расщепляется" на две ветви - в одной из них фотон прошёл через левую щель, а в другой - через правую. Но эти ветви не обязательно становятся независимыми - они могут продолжить жить вместе и интерферировать друг с другом, при условии, что дальнейшая их эволюция приведёт к совместимым результатам. Фотон может даже "не знать", сохраним мы информацию о нём или нет - выбор может быть сделан пост-фактум. И здесь возникают вопросы:

    1. Какой механизм проверяет совместимость двух ветвей и отсеивает варианты, приводящие к нарушению законов физики? (Куда девается вариант, когда частица пролетает через обе щели и оба детектора показывают половинный сигнал?)

    2. Можем ли мы понять, что какие-то наши действия невыполнимы, так как в будущем приведут к квантовому конфликту (например, никак не удаётся провести какой-нибудь опыт)? Можем ли мы почувствовать "притяжение будущего"?


    1. DrSmile
      22.08.2021 22:01
      +3

      Итак, электрон взаимодействует с атомами, из которых состоят края щели, и подвергается дифракции.
      Дифракция происходит не из-за взаимодействия с краями щели (в некотором смысле), а из-за пропажи вариантов электрона, которые втыкаются в стенку. Там, где раньше была деструктивная интерференция с другими путями электрона, теперь этих других путей нет. Т. е. электроны в двухщелевом опыте напрямую не взаимодействуют со стенками, они пролетают свободно.

      Но не суть, есть опыты, где взаимодействие реально есть: например, фотоны с зеркалами. В этом случае принципиально важно, чтобы никакой информации о пути фотона не оставалось в установке, иначе интерференция пропадает. Конкретно для зеркал это значит, что зеркала достаточно жесткие, чтобы импульса фотона не хватало на возбуждение даже одного кванта колебаний, все зеркала закреплены на едином жестком столе так, что импульсы от отражений фотонов от разных зеркал взаимно усреднялись, и этот стол должен быть достаточно массивным, чтобы за время существования нескомпенсированных импульсов он не успел сильно сместиться.

      Но тут важно понимать, что физика — наука непрерывная: невозможно сделать идеальную установку, какая-то часть информации в любом случае просочится. Соответственно, какая-то часть всех электронов/фотонов давать интерференционную картину не будет.

      Какой механизм проверяет совместимость двух ветвей и отсеивает варианты, приводящие к нарушению законов физики?
      Неудачный выбор терминов: нарушение законов — это, например, несохранение энергии. Законы сохранения выполняются в каждой ветке и никакое взаимодействие веток не может привести к их нарушению.

      Отсеивание же целых веток возможно путем деструктивной интерференции. Т. е. два почти идентичных мира (отличающихся состоянием одной частицы) со взаимно противоположными амплитудами могут деструктивно проинтерферировать и взаимоуничтожиться.


    1. Scinolim
      23.08.2021 11:07

      Как многомировая интерпретация может что-то лучше объяснять, если её невозможно вычислить? Это космология какая-то уже, а не квантовая физика. Хорошая правильная теория должна рассчитываться не сложнее законов Ньютона, а не плодить заплатки на каждом факте, не соответствующим расчётам.


    1. DmitriyN
      23.08.2021 15:53
      +1

      Процесс всегда такой - если информация покидает объект и попадает в неконтролируемую среду, то начинается декогеренция. Если нет, то не начинается. Например, в случае со щелями, как справедливо сказал @DrSmile, участвуют в процессе только пути, которые как раз не взаимодействовали. Но это совершенно не обязательно. Например, при дифракции электронов на кристалле, некоторые электроны отражаются упруго (читай, не оставляют следа в решетке) и интерферируют. А некоторые - неупруго (порождают вторичные электроны, фононы и т.п.). Они, в итоге, создают диффузный фон на дифракционном изображении.

      Под покиданием информации нужно понимать запутывание степеней свобды системы и внешней среды. Когда эектрон попадает на экран, он запутывается с квазичастицами в экране, а те, в свою очередь, с электромагнитным полем (на самом деле все несколько сложнее, потому что электрон поглощается). Есть некоторые причины, по которым эти квазичастицы декогерируют таким образом, что их конечные состояния обладают хорошо определенным положением (гуглить einselection), что образует четко выраженный базис, в который система декогерирует после поглощения. Это и приводит к тому, что мы видим вспышку в определенном месте.

      Что касается квантового ластика - механизм, который "проверяет" корректность - это просто законы физики. Половина фотона принципиально не может поглощаться просто благодаря его природе.

      Что касается конфликта - очень вряд ли (хотя я бы полностью не зарекался). Вот например, лагранжева формулировка классической механики тоже не локальна во времени, однако вряд ли кому-то покажется, что благодаря этому есть какие-то действия, невыполнимые из-за будущего конфликта (хотя они есть - те действия, которые нарушают законы физики).


  1. Vsevo10d
    22.08.2021 20:55
    +2

    В частности, они никогда не проявляют волноподобных свойств, которые порождает квантовый интерференционный узор.

    Помнится, нам еще на первом курсе говорили, что макрообъект, если начать считать его длину волны по hc/л=mv2 даст такие маленькие длины волн, что они будут меньше самой мелкой возможной дифракционной решетки (меньше атомов по сути), и ни о каком узоре речи идти не будет.


  1. Sergey_Cheban
    23.08.2021 00:14

    Есть смысл расширить статью, поговорив о времени. Есть подозрение, что рисунок 6 должен быть симметричным, и раздвоенной должна быть не только правая часть (будущее), но и левая (прошлое).


  1. sshmakov
    23.08.2021 08:11

    Я ничего не понимаю в квантовой механике, поэтому не постесняюсь задать вопрос знающим людям.

    Если волновая функция элементарной частицы описывает вероятность ее положения/состояния, а у двух частиц у каждой есть собственная волновая функция, то взаимодействие этих частиц есть взаимодействие двух функций, порождающее новую функцию, у которой график (распределение вероятностей) будет сильно отличаться от графиков первых двух. Появятся состояния, даже регионы состояний, в которых эти две частицы находиться одновременно не могут. Увеличивая количество частиц, мы продолжаем ограничивать им степени свободы. Для макрообъекта ограничения становится такими жесткими, что его положение и движение описывается законами классической физики.

    Это раз.

    Следствие - когда в квантовую систему из одной частицы вводится макрообъект, будь то человек-наблюдатель или склянка с ядом в эксперименте Шредингера, то никакой суперпозиции не возникает, точнее она низводится до взаимодействия макрообъекта с одним из вариантов исхода - так как макрообъект не может находиться в двух состояниях одновременно (см. раз), то все остальные вероятности волновой функции нашей частицы обнуляются, к исходу не приводят, новую вселенную не порождают.

    Это два.

    Да, я понимаю, что это не математический подход, а утилитарный, инженерный. Но я буду благодарен, если кто-то укажет мне на ошибку в рассуждениях.


    1. 0serg
      23.08.2021 09:55

      А тут все довольно просто. Обычно все происходит именно так как Вы описываете. Но экспериментальные установки строятся именно таким образом чтобы распространить состояние волновой функции на макрообъекты. Это очень тонкие и точные приборы специально настроенные таким образом чтобы «наблюдать» коллапс волновой функции, но «наблюдение» — это собственно процесс который переводит макроскопическую экспериментальную установку в наблюдаемо разное макроскопическое состояние в зависимости от разных квантовых исходов.


    1. HappyLynx
      23.08.2021 13:48

      так как макрообъект не может находиться в двух состояниях одновременно (см. раз)

      Вот тут ошибка. Да, у нас есть дискретные квантовые характеристики, например, спин электрона, которые дискретно "расщепляют" вселенную на варианты при взаимодействии этого электрона с чем либо, но так же у нас есть и непрерывные характеристики, например, координата электрона, и мы можем изменять функцию вероятности положения электрона взаимодействиями с ним, и да, в рамках макрообъекта мы получаем гораздо более узкие и выраженные пучности на этой функции, однако, части области определения, где её значение превышает ноль, это все еще непрерывные множества, которые отражают бесконечное и, что важнее, не счетное, множество "миров", просто те из них, в которых макрообьект проявляет квантовые свойства, настолько невероятны, что ни один экспериментатор, проведи он хоть все время жизни вселенной, наблюдая за яблоком, не дождался бы его туннелирования на обратную сторону Юпитера. И тем не менее, вероятность этого события не равна нулю, и с точки зрения ММИ, когда каждый квант света от яблока достигает роговицы наблюдателя, рождается бесконечное несчетное множество тех, кто наблюдает туннелирование яблока к Юпитеру, но отношение мощности этого множества к мощности множества тех, у кого яблоко продолжает лежать на столе, стремится к нулю. И, чтобы меня не закидали помидорами, поясню, человек в качестве наблюдателя взят просто для примера, точно так же можно взять камень, лежащий рядом с яблоком, и каждое взаимодействие яблока с камнем рождает несчетное множество миров, где камень лежит, а яблоко уже у Юпитера, но так же рождает на многие порядки больше тех миров, где яблоко все еще рядом с камнем.

      Т.е. не нужно воспринимать расслоение вселенной из ММИ, как распад на дискретные миры. Это, скорее, появление конуса миров из точки, вне которого вероятность 0, при этом вероятность внутри конуса неоднородна, а для макро объектов настолько неоднородна, что пучности начинают походить на лучи, и тем не менее, это не лучи, т.к. интегрированная вероятность для любого вложенного конуса не равна нулю.


      1. sshmakov
        23.08.2021 15:11

        Спасибо

        в рамках макрообъекта мы получаем гораздо более узкие и выраженные пучности на этой функции, однако, части области определения, где её значение превышает ноль, это все еще непрерывные множества

        А вот эта непрерывность, она как-то подтверждалась математически или экспериментально? И насколько мы сможем различить мир А от мира Б?


        1. HappyLynx
          23.08.2021 15:35

          А вот эта непрерывность, она как-то подтверждалась математически или экспериментально?

          Когда мы говорим о не дискретных квантовых параметрах, навроде координаты в пространстве, то этот параметр задает область определения функции распределения вероятностей, которая получается кусочно-гладкой, а значит есть непрерывные интервалы значений параметра, где вероятность строго больше ноля (т.е. есть миры, реализующие данное значение параметра), а множество значений из непрерывного интервала - множество несчетное.

          И насколько мы сможем различить мир А от мира Б?

          Ровно так же, как можете отличить два действительных числа, сколь мала не была бы дельта между ними. Если технически не сможете различить прямым наблюдением, то косвенно по последствиям. Собственно, об этом как раз и речь в ММИ, расслоение миров происходит именно в момент наступления различимых последствий, причем это локальное расслоение, т.е. пока фотон от туннелирующего яблока (из моего примера выше) летит до наблюдателя человека или камня, в той области, куда он уже долетел, мир распался на те, где яблоко все еще лежит на столе, и те, где оно телепортировалось, при этом мир наблюдателя еще не распался, и яблоко в нем все еще находится в суперпозиции своих состояний.